Samacheer Kalvi 8th Social Science Guide Geography Chapter 4 Migration and Urbanisation

Tamilnadu State Board New Syllabus Samacheer Kalvi 8th Social Science Guide Pdf Geography Chapter 4 Migration and Urbanisation Text Book Back Questions and Answers, Important Questions, Notes.

Tamilnadu Samacheer Kalvi 8th Social Science Solutions Geography Chapter 4 Migration and Urbanisation

Samacheer Kalvi 8th Social Science Migration and Urbanisation Text Book Back Questions and Answers

I. Choose the correct answer.
1. People move from ……………. to ……………. mainly in search of better jobs.
a) Rural to Urban
b) Urban to Rural
c) Hills to plains
d) Plains to hills
Answer:
a) Rural to Urban

2. A person moves from his own country to another country is known as ……………. .
a) Immigrant
b) Refugee
c) Emigrant
d) Asylum seeker
Answer:
c) Emigrant

3. The migration in search of fertile agricultural land is ……………. migration.
a) Rural to Rural
b) Rural to Urban
c) Urban to Rural
d) Urban to Urban
Answer:
a) Rural to Rural

Samacheer Kalvi 9th Social Science Guide Geography Chapter 7 Mapping Skills

4. War is one of the causes of human migration.
a) Demographic
b) Socio-cultural
c) Political
d) Economic
Answer:
c) Political

5. The main reason for the development of urbanisation in the pre-historic period was
a) Production of food grains
b) Domestication of cattle
c) Fishing
d) hunting
Answer:
b) Domestication of cattle

II. Fill in the blanks:
1. Urbanisation is determined by ……………. the number of factors.
Answer:
three

2. ……………. is the major push factor operating in rural areas.
Answer:
Migration

3. ……………. Metropolitan city in India has the second-highest urban population in the world.
Answer:
Delhi

4. The movement of a person based on his free will and desire to live in a better place is called ………… migration.
Answer:
Voluntary

5. In modem time urban growth was accelerated by the development of …………….
Answer:
New trade route

III. Match the following:

1. Emigration a) In-migration
2. Immigration b) Outmigration
3. Pull factor c) Unemployment
4. Push factor d) Socio-cultural migration
5. Marriage e) Employment opportunity

Answer:

1. Emigration b) Outmigration
2. Immigration a) In-migration
3. Pull factor e) Employment opportunity
4. Push factor c) Unemployment
5. Marriage d) Socio-cultural migration

IV. State whether the following statements are true or false.
1. Slums are generally found in cities.
Answer:
True

2. Mass migration is absent in the modem period.
Answer:
True

Samacheer Kalvi 9th Social Science Guide Geography Chapter 7 Mapping Skills

3. The process of urbanisation has a short history.
Answer:
False

4. Cities and towns are the major polluters of the environment.
Answer:
True

5. Transhumance is also referred to as seasonal migration.
Answer:
True

V. Consider the given statements and choose the correct option from the given ones.
1. Statement (A): Urbanisation is mainly due to the movement of people from rural to cities.
Reason (R): Rural to urban migration is not a predominant one.
a) A is correct but R is incorrect
Both A and R are incorrect
c) Both A and R are correct
d) A is incorrect and R is correct
Answer:
c) Both A and R are correct

VI. Answer the following questions in brief:
1. Define “Migration”.
Answer:
Migration has been defined differently by different experts. In general, migration is defined as the permanent or semi permanent change of residence of an individual or group of people over a significant distance. So, the term migration refers to the movement of people from one place to another.

Samacheer Kalvi 9th Social Science Guide Geography Chapter 7 Mapping Skills

2. What are the causes of rural to urban migration?
Answer:
The main causes are

  • Searching for jobs
  • Education
  • Recreation facilities.

3. State the causes of ecological or natural migration.
Answer:

  1. Natural causes include volcanic eruption, mankind changed considerably.
  2. At this stage, people continued to move from one region to another in search of fertile land for cultivation.

4. Name any two pull factors of migration.
Answer:

  • Employment opportunities
  • Education.

5. What is Urbanisation?
Answer:
Urbanization refers to the process in which there is an increase in the proportion of the population living in towns and cities.

Samacheer Kalvi 9th Social Science Guide Geography Chapter 7 Mapping Skills

6. List out any four most populous cities in the world.
Answer:
Tokyo, Delhi, Mexico City, Sao Paulo.

VII. Answer the following questions in detail.
1. What are the different types of migration? Explain.
Answer:
Migration can be classified in several ways. They are:
1. Based on the movement associated with administrative limits.

(a) Internal Migration
(b) International Migration

2. Based on the willingness of the migrants for migration.

(a) Voluntary Migration
(b) Involuntary Migration

3. Based on the duration of stay of migrants in the place of destination.

(a) Short term Migration
(b) Long term Migration
(c) Seasonal Migration

1. Based on the movement associated with administrative limits.
(a) Internal Migration:

  1. The movement of people within a country is known as internal migration.
  2. Further internal migration is classified into four types. They are,
    • Rural to Urban migration
    • Urban to Urban migration
    • Rural to Rural migration and
    • Urban to Rural migration

(b) International Migration:
Migration that occurs across the national boundaries is known as international migration.

2. Based on the Willingness of the migrants for migration:
(a) Voluntary migration:

  1. If the migration takes place on perspus free will, initiative, and desire to live in a better place.

(b) Involuntary migration:

  1. If the migration takes place against the will of migrants is known as involuntary migration.

3. Based on the duration of stay of migrants in the place of destination:
(a) Short term migration:

  1. The migrants stay outside only for a short duration before returning to the place of origin.

(b) Long term migration:

  1. The migrants stay outside at least for a few years.

(c) Seasonal migration:

  1. Usually, a group of people migrates from their native places during a particular season and return after end of that season.
  2. Transhumance is another example of seasonal migration.

2. Explain in detail the various causes of migration?
Answer:

  • The favourable factors which attract people towards a location are called pull factors.
  • The unfavourable factors which make the people move out from a location are called push factors.
  • Human migration can be categorized under five groups are
    • Ecological or natural causes of migration
    • Economic causes of migration
    • Socio-cultural causes of migration
    • Demographic causes of migration
    • Political causes of migration.

3. Discuss the problems of urbanisation.
Answer:

1. Housing and Slums:

  • There is a lack of space for housing and a marked reduction in the quality of housing in the urban areas due to increase in population.

2. Over Crowding:

  • Over – crowding leads to unhealthy environment in the urban areas.
  • It also the cause of many diseases and riots.

3. Water Supply, Drainage and Sanitation:

  • No city has round a clock water supply in the world.
  • The drainage situation is equally bad.
  • The removal of garbage is a Himalayan task for urban local bodies.

4. Transportation and Traffic:

  • The absence of planned and adequate arrangements for traffic and transport is another problem in urban centers.
  • The increasing number of two-wheeler and cars make the traffic problem worse.

5. Pollution:

  • Towns and cities are the major polluters of the environment.
  • Several cities discharge their entire sewage and industrial effluents untreated into the nearby rivers. Industries in and around the urban centers pollute the atmosphere with smoke and toxic gases.

IX. Activities
1. List out and analyze the reason for the migration of people in your locality.
Answer:
People migrate for a number of reasons. The reasons and causes for migration would normally fall under these areas:

  1. Environmental – Better climate, calamities, and natural disasters are examples of environmental causes or reasons.
  2. Economic – Moving to find work or moving to follow a particular career path is an example of economic cause or reason.
  3. Cultural – Religious freedom and education is an example of cultural cause or reason.
  4. Political – Civil war or escaping from political persecution is an example of a political cause.
  5. Social – Moving for a better quality of life or moving closer to a family member or friend is an example of a social cause or reason.

2. Collect the pictures and information regarding the effects of urbanization and make an album.
Answer:

  • Slums and Housing
  • Overcrowding
  • Transportation and traffic
  • Pollution.

Samacheer Kalvi 8th Social Science Migration and Urbanisation Additional Important Questions and Answers

I. Choose the Correct answer.
1. The movement of an individual or group to the new country is termed as ………………..
a) Destination
b) Migration
c) Emigrant
d) Immigrant
Answer:
a) Destination

2. Migration is more in ………………. region of the world population.
a) Japan
b) Asia
c) Australia
d) Africa
Answer:
b) Asia

Samacheer Kalvi 9th Social Science Guide Geography Chapter 7 Mapping Skills

3. In 2017, ………………. was the largest country of origin of international migrants.
a) India
b) America
c) Delhi
d) Asia
Answer:
a) India

4. In ………………., the global urban population exceeded the global rural population.
a) 2005
b) 2006
c) 200
d) 2008
Answer:
c) 2007

5. During the beginning of the ………………. itself many cities were found near the Aegean sea.
a) 6th century
b) 7th century
c) 8th century
d) 5th century
Answer:
b) 7th century

6. In the ancient period the migration was mainly for ………………..
a) Environment
b) Health
c) Food
d) Place
Answer:
c) Food

7. The industrial revolution in the century accelerated the growth of towns and cities.
a) 17th
b) 18th
c) 19th
d) 20th
Answer:
c) 19th

Samacheer Kalvi 9th Social Science Guide Geography Chapter 7 Mapping Skills

8. In 1950, ………………. of the world’s population was urban.
a) 20%
b) 30%
c) 40%
d) 50%
Answer:
b) 30%

9. ………………. is the world’s Highest Population city.
a) Masco (Russia)
b) Shangai (China)
c) Delhi (India)
d) Tokyo (Japan)
Answer:
d) Tokyo (Japan)

10. Migration that occurs the national boundaries are known as
a) Internal migration
b) Voluntary migration
c) International migration
d) Involuntary migration
Answer:
c) international migration

II. Fill in the blank:
1. ………………. is a form of geographical mobility of population between a geographical unit to another.
Answer:
Migration

2. Climatic disasters are ………………. factors.
Answer:
push

3. The movement of people within a country is known as ………………..
Answer:
Internal migration

4. If the migration takes place against the will of migrants is known as ………………..
Answer:
Involuntary migration

5. The migration of agricultural workers during Saving sources is known as ………………..
Answer:
Seasonal migration

6. ………………. is another example of Seasonal migration.
Answer:
Transhumance

7. The number of international migrants in 2017 was ………………..
Answer:
258 million

8. Skilled people from economically backward countries move to developed Countries called ………………..
Answer:
Backwash effect

9. ………………. is the process in which there is an increase in the proportion of population living in towns and cities.
Answer:
Urbanisation

Samacheer Kalvi 9th Social Science Guide Geography Chapter 7 Mapping Skills

10. The excess production of ………………. was the major reason for urbanisation.
Answer:
foodgrains

11. The ………………. revolution in the 19th century accelerated the growth of towns and cities.
Answer:
industrial

12. Rapid rate of urbanisation results in the development of ………………..
Answer:
Slums

13. In ………………., ………………. regions migrants are predominantly men.
Answer:
Asia, Africa

14. The ………………. factors which attract people towards a location.
Answer:
favourable

15. ………………. is a natural cause of migration.
Answer:
Ecology

III. Match the following:

1. Natural cause a) War
2. Economic cause b) Social Discrimination
3. Socio-cultural Cause c) Unemployment
4. Political Cause d) Scarcity of food

Answer:

1. Natural cause d) Scarcity of food
2. Economic cause c) Unemployment
3. Socio-cultural Cause b) Social Discrimination
4. Political Cause a) War

IV. State whether the following statements are true or false.
1. The number of vehicles increase is not the cause of air pollution.
Answer:
False

2. The favourable factors which attract people towards a location.
Answer:
True

3. The movement of people within a country is known as internal migration.
Answer:
True

4. Migration develops both the areas of origin of migration and the areas of destination.
Answer:
False

Samacheer Kalvi 9th Social Science Guide Geography Chapter 7 Mapping Skills

5. After the 11th century, the European Countries, increased their overseas trade.
Answer:
True

V. Consider the given statements and choose the correct option from the given ones.
1. Statement: Migration may influence the occupational Structure of the Population of an area through this it will certainly affect the economy of the regions also.
Reason: Brain drain refers to the migration in which skilled people from economically backward countries move to developed countries in search of better opportunities,
a) A is correct but R is incorrect
b) Both A and R are incorrect
c) Both A and R are correct
d) A is incorrect and R is correct
Answer:
c) Both A and R are correct

2. Assertion: At the end of the 13th century, Paris London, Geneva, Milan, and Venice were the important cities found in Europe.
Reason: The European countries increased their overseas trade.
a) A is correct but R is incorrect
b) Both A and R are incorrect
c) Both A and R are correct
d) A is incorrect and R is correct
Answer:
c) Both A and R are correct

3. i) Migration that occurs across the national boundaries are known as Internal migration.
ii) If the migration takes place of a person’s free will is known as voluntary migration.
iii) The migration of agricultural workers during Sowing Seasons is known as Seasonal migration.
iv) The migration from rural areas to growing towns and cities mainly in search of employment, education, and recreation facilities.
a) i, iv correct
b) ii correct,
c) ii and iv correct
d) all are correct
Answer:
c) ii and iv correct

VI. Answer the following questions in brief.
1. What are the two factors of migration?
Answer:

  • The favourable factors which attract people towards a location are called pull factors.
  • The unfavourable factors which make the people to move out from a location are called push factors.

2. What is Backwash Effect?
Answer:

  • Brain drain refers to the migration in which skilled people from economically backward countries move to developed countries in search of better opportunities.
  • Eventually, this leads to backwardness in source regions. This is called a “backwash effect”.

3. What are the Environmental Consequences that occured during Migration?
Answer:
The overpopulation in urban areas leads to the pollution of air, water, and soil. Scarcity of drinking water, lack of space for housing, traffic congestions, and poor drainage are the common environmental problems that prevail in urban areas.

Samacheer Kalvi 9th Social Science Guide Geography Chapter 7 Mapping Skills

4. What are the causes of urbanisation?
Answer:

  • Urbanisation is driven by three factors: natural population growth, rural to urban migration, and the reclassification of rural areas into urban areas.
  • Present-day urbanisation includes changes in demographics, land cover, economic processes, and characteristics of geographic area.

5. What are the noted Prehistoric cities of the world?
Answer:

  • Ur and Babylon in Mesopotamia
  • Thebes and Alexandria in Egypt
  • Athens in Greece
  • Harappa and Mohenjodaro in India were noted prehistoric cities of the world.

6. What are the major cities in Africa?
Answer:
Major cities in Africa are Cairo, Nairobi, Mombasa, Bulawayo, Duala, Abidian, Logos, Accra, Addis Abba, Leopoldville, Luanda, Cape Town, Natal, Pretoria.

VII. Distinguish.
1. Short term migration and Long term migration.

Short term migration Long term migration
1. In this kind of migration, the migrants stay outside only for a short duration before returning to the place of origin. It is a kind of migration in which the migrants stay outside at least for a few years.
2. The duration may be from a few days to a few months. The migrants stayed a long time.

VIII. Answer the following questions in detail.
1. Explain Pull factors and Push factors of migration.

Pull factors Push factors
1. Least hazard-prone zones. Hazard prone zones.
2. Favourable climate. Climate change (including extreme weather events).
3. Abundance of natural resources and minerals (e. g. water, oil, uranium). Crop failure and scarcity of food.
4. Social Unification. Social Discrimination.
5. Under population. Overpopulation.
6. Political security. War, civil, unrest.

2. What are the Consequences of Migration?
a) Demographic consequences:

  • It changes the age and sex composition of the population.
  • Migration of females after their marriage leads to a decline in sex ratio in the source regions and increases in the sex ratio in the regions of destinations.
  • The migration of male workers in search of jobs decreases the independent population of the source regions which increases the dependency ratio.

b) Social consequences:

  • The migration of people from different regions towards an urban area leads to the formation of a plural society.
  • It helps the people to come out of narrow mindedness and people become generous.

c) Economic consequences:

  • The migration of more people from overpopulated to underpopulated regions results in the imbalance of the resource-population ratio.
  • In some cases, the regions of over and under population may become the regions of optimum population.
  • Migration may influence the occupational structure of the population of an area.
  • Through this, it will certainly affect the economy of the regions also.
  • Brain drain is a consequence of migration.
  • Brain drain refers to the migration in which skilled people from economically backward countries move to developed countries in search of better opportunities
  • Eventually, this leads to backwardness in source regions. This is called a “backwash effect”.

d) Environmental consequences:

  • Large scale movement of people from rural to urban areas causes overcrowding in cities and puts heavy pressure on resources.
  • It leads to the rapid growth of cities.
  • The overpopulation in urban areas leads to the pollution of air, water, and soil.
  • Scarcity of drinking water, lack of space for housing, traffic congestion, and poor drainage are the common environmental problems that prevail in urban areas.
  • The lack of space for housing and the rising land cost lead to the formation of slums.

Samacheer Kalvi 8th Social Science Guide Geography Chapter 3 Hydrologic Cycle

Tamilnadu State Board New Syllabus Samacheer Kalvi 8th Social Science Guide Pdf Geography Chapter 3 Hydrologic Cycle Text Book Back Questions and Answers, Important Questions, Notes.

Tamilnadu Samacheer Kalvi 8th Social Science Solutions Geography Chapter 3 Hydrologic Cycle

Samacheer Kalvi 8th Social Science Hydrologic Cycle Text Book Back Questions and Answers

I. Choose the Correct Answer.
1. The process in which the water moves between the oceans, atmosphere and land is called
a) River Cycle
b) Hydrologic Cycle
c) Rock Cycle
d) Life Cycle
Answer:
b) Hydrologic Cycle

2. The percentage of fresh water on the earth is
a) 71
b) 97
c) 2.8
d) 0.6
Answer:
c) 2.8

3. The process of changing of water from gaseous to liquid form is known as
a) Condensation
b) Evaporation
c) Sublimation
d) Rainfall –
Answer:
a) Condensation

4. Water that flows in the sub-soil or through the ground into the streams, rivers, lakes and oceans is termed as
a) Condensation
b) Evaporation
c) Transpiration
d) Runoff
Answer:
d) Runoff

Samacheer Kalvi 9th Social Science Guide Geography Chapter 7 Mapping Skills

5. The evaporation of water from the leaves of plants is called
a) Transpiration
b) Condensation
c) Water vapour
d) Precipitation
Answer:
a) Transpiration

6. Water that is good enough to drink is called
a) Groundwater
b) Surface water
c) Potable water
d) Artesian water
Answer:
c) Potable water

II. Fill in blank:
1. The degree of water vapour present in the atmosphere is known as …………………
Answer:
weather

2. There are ………………… phases in the water cycle.
Answer:
Three

3. The falling of water towards the earth surface from the atmosphere in any form is known as …………………
Precipitation

4. The precipitation with a rain drop size of <0.5mm in diameter is known as ………………….
Answer:
drizzle

5. Mist is denser than ………………….
Answer:
Clouds

III. Match the following:

1. Vegetation a) Clouds
2. Condensations b) Sleet
3. Snow and raindrops c) At the surface
4. Infiltration d) Transpiration

Answer:

1. Vegetation d) Transpiration
2. Condensations a) Clouds
3. Snow and raindrops b) Sleet
4. Infiltration c) At the surface

IV. Choose the correct statement:
1. Evaporation refers to
I. The process in which the gaseous form of water changes into liquid form.
II. It refers to the process in which the liquid form of water changes into a gaseous form.
III. Water boils at 100°C temperature but, it actually begins to evaporate at 0°C.
IV. It is responsible for the formation of clouds.
a) I, IV, and V are correct
b) II only correct
c) II and III are correct
d) All are correct
Answer:
c) II and III are correct

V. State whether thefollowing^statements are True or False:
1. Water boils at 212°F temperature but, it begins to evaporate at 32°F.
Answer:
True

2. Mist is not the tiny droplets of water hanging in the air.
Answer:
False

Samacheer Kalvi 9th Social Science Guide Geography Chapter 7 Mapping Skills

3. The sub-surface runoff is usually referred to as interflow.
Answer:
True

VI. Answer in brief:
1. Write a short note on the aquifer.
Answer:
An aquifer is an underground layer of water-bearing rock.

2. Define “ hydrological cycle”.
Answer:
The hydrological cycle is a global sun-driven process where water is transported from oceans to the atmosphere, from the atmosphere to land, and from land back to oceans

3. How is the dew formation takes place?
Answer:
Dew is a water droplet formed by the condensation of water vapour on a relatively cold surface of an object. It forms when the temperature of an object drops below the dew point temperature.

4. Write a short note on surface run-off.
Answer:
Surface Runoff is the portion of rainfall, which enters the stream immediately after the rainfall. It occurs when the rainfall is longer, heavier, and exceeds the rate of infiltration. In this condition, the excess water makes a head over the ground surface, which tends to move from one place to another following land gradient and is known as overland flow. When the overland flow joins the streams, channels or oceans, it is termed as surface runoff or surface flow.

VII. Give reasons:
1. Infiltration of water is low in the region of non-porous soil.
Answer:
Because percolation moves the infiltrated water through the soil profile and rock layers to form groundwater.

2. Freshwater is less on the earth.
Answer:
Most of the water on the earth is saline and is found in seas and oceans, which constitutes about 97.2%.

Samacheer Kalvi 9th Social Science Guide Geography Chapter 7 Mapping Skills

3. Snowfall is common in the polar region and mountainous regions.
Answer:
Often water vapour in a cloud is converted directly into snow pieces due to lowering of temperature and so it is common in the polar and mountainous regions.

VIII. Answer in a paragraph:
1. Explain the different stages involved in the hydrological cycle.
Answer:
The three important phases of the hydrologic cycle are:

  • Evapotranspiration,
  • Precipitation and
  • Runoff.

1. Evapotranspiration:
It is defined as the total loss of water from the earth through evaporation from the surface water bodies and the transpiration from vegetation. In cropped areas, it is difficult to determine the evaporation and transpiration separately. Therefore it is collectively called evapotranspiration.

2. Precipitation:
It refers to all forms of water that fall from clouds and reaches the earth’s surface. For the occurrence of precipitation, cloud droplets or ice crystals must grow heavy enough to fall through the air. When the droplets grow large in size, they tend to all. While moving down, by collecting some small droplets, they become heavy enough to fall out of the cloud as raindrops.

3. Runoff:
Runoff is the water that is pulled by gravity across the land’s surface. It replenishes groundwater and surface water as it percolates into an aquifer (it is an underground layer of water-bearing rock) or moves into a river, stream, or watershed. It comes from unabsorbed water from rain, snowmelt, irrigation or other sources, comprising a significant element in the water cycle as well as the water supply when it drains into a watershed. Runoff is also a major contributor to erosion which carves out canyons, gorges, and related landforms.

2. Distinguish between evaporation and transpiration.

Water boils at 100°C temperature but, it actually begins to evaporate at 0°c. Much of the water taken up by the plants is released through transpiration.

Evaporation Transpiration
1. The process in which the liquid form of water changes into gaseous form. The process by which the water content in the plant are released into the atmosphere in the form of water vapour.
3. The amount of water gets evaporated is about the same as the amount of water delivered to earth as precipitaion. The soil water content and the ability of the soil to conduct water the roots and the nature of the plant parts also determine the transpiration rate.
4. The rate of evaporation is low during the periods of calm winds than during windy times. The rate of transpiration is affected by the temperature, wind and humidity.

3. Give a detailed explanation on different forms of precipitation.
The form of precipitation in a region depends on the kind of weather or the climate of the region. The Precipitation in the warmer parts of the world is always in the form of rain or drizzle.
In colder region, precipitation may fall as snow or ice.
Common types of precipitation include rain, sleet, freezing rain, hail & snow.

Rain:

  • The most common kind of precipitation is rain.
  • The precipitation in the form of water droplet is rain.
  • Below 0.5 mm in diameter is known as drizzle and above 0.5 mm in diameter is known as rain.

Sleet:
The precipitation which takes place in the form of a mixture of water droplets and tiny particles of ice is known as sleet.

Freezing Rain:

  • Raindrops falling through cold air near the ground do not freeze in the air.
  • Instead, the raindrops freeze when they touch a cold surface.
  • This is called freezing rain.

Hail:
The precipitation which consists of round pellets of ice which are larger than 5mm in diameter is called as hail or hailstones.
Snow:
The precipitation in form of a powdery mass of ice is known as snowfall which is common in the polar and high mountainous regions.

Samacheer Kalvi 9th Social Science Guide Geography Chapter 7 Mapping Skills

4. Explain the run-off and its types.

Runoff is the water that is pulled by gravity across the land’s surface. It replenishes groundwater and surface water as it percolates into an aquifer (it is an underground layer of water-bearing rock) or moves into a river, stream, or watershed.

Types of Runoff:
Based on the time interval between the instance of rainfall and generation of runoff, the runoff may be classified into the following three types

1. Surface Runoff:
It is the portion of rainfall, which enters the stream immediately after the rainfall. It occurs when the rainfall is longer, heavier, and exceeds the rate of infiltration. In this condition, the excess water makes a head over the ground surface, which tends to move from one place to another following a land gradient and is known as overland flow. When the overland flow joins the streams, channels, or oceans, it is termed as surface runoff or surface flow.

2. Sub – Surface Runoff:
The water that has entered the subsoil and moves laterally without joining the water-table to the streams, rivers or oceans is known as sub-surface runoff. The subsurface runoff is usually referred as interflow.

3. Base Flow:
It is a flow of underground water from a saturated groundwater zone to a water channel. It usually appears at a downstream location where the channel elevation is lower than the groundwater table. Groundwater provides the streamflow during dry periods of small or no precipitation.

Samacheer Kalvi 8th Social Science Hydrologic Cycle Additional Important Questions and Answers

I. Choose the Correct answer.
1. The process of change of water into gaseous form is called ………………….
a) Evaporation
b) Evapotranspiration
c) Precipitation
d) Condensation
Answer:
a) Evaporation

2. …………… is the prime element which affects the rate of evaporation.
a) Temperature
b) Rain
c) Air
d) Mist
Answer:
c) Air

3. The quantity of water present on the earth is about ………………. million cubic miles.
a) 236
b) 263
c) 326
d) 362
Answer:
c) 326

4. …………………. Saturation occurs when the temperature drops down.
a) Air
b) Water
c) Land
d) Soil
Answer:
a) Air

5. ……………… of the moisture form plants through transpiration.
a) 10%
b) 50%
c) 80%
d) 90%
Answer:
a) 10%

Samacheer Kalvi 9th Social Science Guide Geography Chapter 7 Mapping Skills

6. The evaporation is more than precipitation in the …………………..
a) Oceans
b) Land side
c) Desert land
d) Water Source
Answer:
a) Oceans

7. The process of change of gaseous form of water into liquid form is called as ______________
a) Vapourisation
b) Transpiration
c) Condensation
d) Infiltration
Answer:
c) Condensation

8. An air layer next to the earth’s Surface to 1000 m or lower is called as _____________
a) Dew
b) Frost
c) Fog
d) Mist
Answer:
c) Fog

9. Water entering the soil at the Surface of the ground is termed as _____________
a) Infiltration
b) Percolation
c) Condensation
d) Vaporisation
Answer:
a) Infiltration

10. The flow of water from the unsaturated zone to the Saturated zone is termed as
a) Infiltration
b) Percolation
c) Condensation
d) Vaporisation
Answer:
b) Percolation

II. Fill in the blanks:
1. Water in India is available in Sources.
Answer:
Three

2. Hydrology is the science which deals with the various aspects of water such as ………………, ……………… and ……………….
Answer:
Occurrence, distribution, movement

3. Hydrologic cycle is a global …………….. process.
Answer:
Sun-driven

4. ………………… refers to all forms of water that fall from clouds and reaches the earth’s Surface.
Answer:
Precipitation

Samacheer Kalvi 9th Social Science Guide Geography Chapter 7 Mapping Skills

5. The three important phases of the hydrologic cycle are …………….., ………………., and ……………..
Answer:
Evapotranspiration, Precipitation, Run off

6. The Sleet falls, the raindrops freeze into solid particles of ………………….
Answer:
ice

7. Most of the water on the earth is …………… in the surface water.
Answer:
Saline

8. ……………….. is available in the form of glaciers.
Answer:
2.15%

9. ……………….. is a continuous process and takes place naturally.
Answer:
Hydrologic cycle

10. Water boils at
Answer:
100PC

11. The water content in salt water oceans is
Answer:
97.2%

12. The amount of Icecaps and glaciers in Fresh water is
Answer:
68.7%

13. The Ground water content in Fresh water is
Answer:
30.1%

14. The boiling point of water is
Answer:
2129F

15. There is a relationship between temperature and evaporation.
Answer:
positive

III. Match the follpwing:

1. Runoff a) Rain
2. Precipitation b) Snow
3. Polar region c) Water springs
4. Infiltration d) less than 0.5mm
5. Drizzle e) river

Answer:

1. Runoff e) river
2. Precipitation a) Rain
3. Polar region b) Snow
4. Infiltration c) Water springs
5. Drizzle d) less than 0.5mm

IV. Choose the correct statement:
1. a) Water entering the soil at the surface of the ground is termed as infiltration
b) Rain water Soaks into the ground, through the soil and underlying rock layers.
c) Some of the water remains underground and is called ground water.
d) The terms infiltration and percolation are other used inter changeably.
a) I, IV, and V are correct
b) II only correct
c) II and III are correct
d) All are correct
Answer:
d) AH are correct

V. State whether the following statements are True or False:
1. Water in India is available in three sources.
Answer:
True

2. Rain is the tiny droplets of water hanging in the air.
Answer:
False

Samacheer Kalvi 9th Social Science Guide Geography Chapter 7 Mapping Skills

3. Saturation occurs when the temperature increases.
Answer:
False

4. Evaporation is the process in which the liquid form of water changes into a gaseous form.
Answer:
True

VI. Answer in brief:
1. What are the Sources of water available in India.
Answer:

  • Water in India is available in three sources.
  • They are precipitation, surface water, and groundwater.

2. What is hydrology?
Answer:
Hydrology is the science which deals with the various aspects of water such as its occurrence, distribution, movement and properties on the planet earth.

3. What are the factors that increase the rate of evaporation?
Answer:

  • Increase in wind speed
  • Increase in temperature
  • Decrease in humidity and
  • Increase in the areal extent of surface water bodies.

VII.Give reasons:
1. Availability of water on the earth is not uniform. Why?
Answer:
The availability of water on the earth is not uniform because Some places are very rich and some places are poor in water resources.

2. Distinguish between Frost and Mist.
Answer:

Frost Mist
1. The ice crystals formed by the deposition of water vapour on a relatively cold surface of an object are known as frost. Mist is the tiny droplets of water hanging in the air.
2. Density increased Mist is less dense than fog.

VIII. Answer in a paragraph:

1. Water on the Earth – Explain.
Answer:

  • About 71 % of the earth’s surface is covered by water.
  • The quantity of water present on the earth is about 326 million cubic miles.
  • It is hard to visualise this massive quantity of water.
  • Most of the water on the earth is saline and is found in seas and oceans.
  • The salt water constitutes about 97.2% and the fresh water is only about 2.8%.
  • Out of this 2.8%, about 2.2% is available as surface water and the remaining 0.6% is groundwater. From this 2.2% of surface water, 2.15% is available in the form of glaciers and icecaps, 0.01% in lakes and streams and the remaining 0.04% is in other forms.
  • Only about 0.25% of the total ground water of 0.6% can be economically extracted with the present drilling technology.

2. Explain the forms of condensation.
Answer:
Dew, fog, and Clouds are the three major forms of condensation.

  • Dew: It is a water droplet formed by the condensation of water vapour on a relatively cold surface of an object.
  • Frost: The ice crystals formed by the deposition of water vapour on a relatively cold surface of an object is known as frost.
  • Fog: Fog is the suspended tiny water droplets or ice crystals in an air layer next to the earth’s surface.
  • Mist: Mist is the tiny droplets of water hanging in the air.
  • Clouds: Clouds consist of tiny water droplets/ice particles which are so small and light in weight.

Samacheer Kalvi 8th Maths Guide Chapter 7 Information processing InText Questions

Tamilnadu State Board New Syllabus Samacheer Kalvi 8th Maths Guide Pdf Chapter 7 Information processing InText Questions Text Book Back Questions and Answers, Notes.

Tamilnadu Samacheer Kalvi 8th Maths Solutions Chapter 7 Information processing InText Questions

Recap (Text Book Page No. 232 & 233)

Question 1.
Find the number of all possible triangles that can be formed from the triangle given below.
Samacheer Kalvi 8th Maths Guide Chapter 7 Information Processing InText Questions 1Answer:
Single Triangles ⇒ 5
Combination of 2 ⇒ 4
Combination of 3 ⇒ 2
Big triangle ⇒ 1
= 12

Samacheer Kalvi 8th Maths Guide Chapter 7 Information processing InText Questions

Question 2.
Use the numbers given in the figure to form a 3 × 3 magic square.
Samacheer Kalvi 8th Maths Guide Chapter 7 Information Processing InText Questions 2
Answer:
Samacheer Kalvi 8th Maths Guide Chapter 7 Information Processing InText Questions 3

Question 3.
Convert the tree diagram into a numeric expression.
Samacheer Kalvi 8th Maths Guide Chapter 7 Information Processing InText Questions 4
Answer:
(10 × 5) + (9 × 4)
= 50 + 36
= 86

Samacheer Kalvi 8th Maths Guide Chapter 7 Information processing InText Questions

Question 4.
(i) Find the total time taken by the bus to reach from A to E via B , C and D.
(ii) Find which is the shortest route from A to E.
Samacheer Kalvi 8th Maths Guide Chapter 7 Information Processing InText Questions 5
Answer:
(i) Route ⇒ A → B → C → D → E
Time taken ⇒ (7 + 5 + 3 + 6) hrs = 21 hrs

(ii)

Available roots Time taken
(a) A → B → C → D → E 7 + 5 + 3 + 6 = 21 hrs
(b) A → B → D → E 7 + 4 + 6  = 17 hrs
(c) A → B → C → E 7 + 5 + 8 = 20 hrs

Shortest route ⇒ A → B → D → E

Samacheer Kalvi 8th Maths Guide Chapter 7 Information processing InText Questions

Question 5.
Connect the Fibonacci squares through diagonals by curve from corner to corner across each square to draw a Golden Spiral.
Samacheer Kalvi 8th Maths Guide Chapter 7 Information Processing InText Questions 6
Fibonacci Squares

Question 6.
When you plan to buy a shirt, one shop offers a discount of ₹ 200 on MRP ₹ 1000 and another shop offers 15% discount on the same MRP. Where would you buy?
Answer:
Price in 1st shop = ₹ 1000 – ₹ 200 = ₹800
Samacheer Kalvi 8th Maths Guide Chapter 7 Information Processing InText Questions 7
In shop 1.

Samacheer Kalvi 8th Maths Guide Chapter 7 Information processing InText Questions

Question 7.
Amazing park is offers a package deal of 5 entrance passes for ₹ 130. If one entrance pass normally costs ₹ 30, how much will you save by taking advantage of this special deal?
Answer:
Cost of one entrance pass = ₹ 30
∴ Cost of 5 entrance passes = ₹ 5 × 30
= ₹ 150
But special deal price = ₹ 130
Amount of saving = 150 – 130 = ₹ 20

Activity (Text Book Page No. 237)

Question 1.
Determine the number of two digit numbers that can be formed using the digits 1, 3 and 5 with repetition of digits allowed.
The activity consists of two parts
(i) Choose a one’s digit.
(ii) Choose a ten’s digit.
Complete the table given beside
Samacheer Kalvi 8th Maths Guide Chapter 7 Information Processing InText Questions 8
Answer:
Samacheer Kalvi 8th Maths Guide Chapter 7 Information Processing InText Questions 9

Samacheer Kalvi 8th Maths Guide Chapter 7 Information processing InText Questions

Question 2.
Find the three digit numbers that can be formed using the digits 1, 3 and 5 without repetition of digits.
Complete the tree diagram given below to the numbers
Samacheer Kalvi 8th Maths Guide Chapter 7 Information Processing InText Questions 10
Answer:
Samacheer Kalvi 8th Maths Guide Chapter 7 Information Processing InText Questions 11

Samacheer Kalvi 8th Maths Guide Chapter 7 Information processing InText Questions

Activity (Text Book Page No. 242)

Question 1.
Samacheer Kalvi 8th Maths Guide Chapter 7 Information Processing InText Questions 12
(i)
Samacheer Kalvi 8th Maths Guide Chapter 7 Information Processing InText Questions 13
(ii)
Samacheer Kalvi 8th Maths Guide Chapter 7 Information Processing InText Questions 14
(iii)
Samacheer Kalvi 8th Maths Guide Chapter 7 Information Processing InText Questions 15
Answer:
Samacheer Kalvi 8th Maths Guide Chapter 7 Information Processing InText Questions 15

Samacheer Kalvi 8th Maths Guide Chapter 7 Information processing InText Questions

Question 2.
Samacheer Kalvi 8th Maths Guide Chapter 7 Information Processing InText Questions 17
Answer:
(i)
Samacheer Kalvi 8th Maths Guide Chapter 7 Information Processing InText Questions 16
(ii)
Samacheer Kalvi 8th Maths Guide Chapter 7 Information Processing InText Questions 18
(iii)
Samacheer Kalvi 8th Maths Guide Chapter 7 Information Processing InText Questions 19
Answer:
Samacheer Kalvi 8th Maths Guide Chapter 7 Information Processing InText Questions 18

Samacheer Kalvi 8th Maths Guide Chapter 7 Information processing InText Questions

Question 3.
Samacheer Kalvi 8th Maths Guide Chapter 7 Information Processing InText Questions 20
(i)
Samacheer Kalvi 8th Maths Guide Chapter 7 Information Processing InText Questions 21
(ii)
Samacheer Kalvi 8th Maths Guide Chapter 7 Information Processing InText Questions 22
(iii)
Samacheer Kalvi 8th Maths Guide Chapter 7
Answer:
Samacheer Kalvi 8th Maths Guide Chapter 7 Information Processing InText Questions 22

Samacheer Kalvi 8th Maths Guide Chapter 7 Information processing InText Questions

Question 4.
Samacheer Kalvi 8th Maths Guide Chapter 7 Information Processing InText Questions 24
(i)
Samacheer Kalvi 8th Maths Guide Chapter 7 Information Processing InText Questions 25
(ii)
Samacheer Kalvi 8th Maths Guide Chapter 7 Information Processing InText Questions 26
(iii)
Samacheer Kalvi 8th Maths Guide Chapter 7 Information Processing InText Questions 27
Answer:
Samacheer Kalvi 8th Maths Guide Chapter 7 Information Processing InText Questions 25

Samacheer Kalvi 8th Maths Guide Chapter 7 Information processing InText Questions

Question 5.
Samacheer Kalvi 8th Maths Guide Chapter 7 Information Processing InText Questions 28
(i)
Samacheer Kalvi 8th Maths Guide Chapter 7 Information Processing InText Questions 29
(ii)
Samacheer Kalvi 8th Maths Guide Chapter 7 Information Processing InText Questions 30
(iii)
Samacheer Kalvi 8th Maths Guide Chapter 7 Information Processing InText Questions 31
Answer:
Samacheer Kalvi 8th Maths Guide Chapter 7 Information Processing InText Questions 30

Samacheer Kalvi 8th Maths Guide Chapter 7 Information processing InText Questions

Try These (Text Book Page No. 243)

Question 1.
Find any five SETs among these set of cards (repetition of cards allowed).
a.
Samacheer Kalvi 8th Maths Guide Chapter 7 Information Processing InText Questions 32

b.
Samacheer Kalvi 8th Maths Guide Chapter 7 Information Processing InText Questions 33

c.
Samacheer Kalvi 8th Maths Guide Chapter 7 Information Processing InText Questions 34

d.
Samacheer Kalvi 8th Maths Guide Chapter 7 Information Processing InText Questions 35

e.
Samacheer Kalvi 8th Maths Guide Chapter 7 Information Processing InText Questions 36

f.
Samacheer Kalvi 8th Maths Guide Chapter 7 Information Processing InText Questions 37

g.
Samacheer Kalvi 8th Maths Guide Chapter 7 Information Processing InText Questions 38

h.
Samacheer Kalvi 8th Maths Guide Chapter 7 Information Processing InText Questions 39

i.
Samacheer Kalvi 8th Maths Guide Chapter 7 Information Processing InText Questions 40

j.
Samacheer Kalvi 8th Maths Guide Chapter 7 Information Processing InText Questions 41
Answer:
(i) (a), (c), (h)
(ii) (e), (g), (j)
(iii) (a), (b), (d)
(iv) (f), (i), (j)
(v) (f), (i), (h)

Samacheer Kalvi 8th Maths Guide Chapter 7 Information processing InText Questions

Question 2.
This is an example for a magic square in SETs. Can you make another two?
Samacheer Kalvi 8th Maths Guide Chapter 7 Information Processing InText Questions 42
Answer:
Repeat one square with every thing 2 another square with everything 3.

Activity (Text Book Page No. 250)

Using the given Table I, find the pattern, answer the following questions and colour the values in the given Table Il. One is done for you.
Table – 1
Samacheer Kalvi 8th Maths Guide Chapter 7 Information Processing InText Questions 43
Question 1.
Where are the even Fibonacci Numbers?
Colour both the term n and where F(n) is even in yellow.
Do you find any pattern?
Every Third Fibonacci number is a multiple of 2(even).
Answer:
i.e., a multiple of F(3) or 2 = F(3)

Samacheer Kalvi 8th Maths Guide Chapter 7 Information processing InText Questions

Question 2.
Where there are Fibonacci numbers which are multiple of 3?
Colour both the term n and where F(n) is multiple of 3 in red.
Write down the pattern you find
Every 4th Fibonacci number is a multiple of 3.
Answer:
i.e., a multiple of F(4) or 3 = F(4)

Question 3.
What about the multiple of 5?
Colour both the term n and where F(n) is multiple of 5 in blue. ‘
Write down the pattern you find.
Every …………………………… i.e ………………………………
Answer:
Every 5th Fibonacci number i.e a multiple of F(5) or 5 = F(5)

Question 4.
What about the multiple of 8?
Colour both the term n where F(n) is multiple of 8 in green.
Write down the pattern you find.
Every …………………………… i.e ………………………………
Answer:
Every 6th Fibonacci number i.e a multiple of F(6) or 8 = F(6)

Samacheer Kalvi 8th Maths Guide Chapter 7 Information processing InText Questions

Table 2.
Samacheer Kalvi 8th Maths Guide Chapter 7 Information Processing InText Questions 44
From the above activity, we conclude that
Every Fibonacci number is a factor of (a term number of) Fibonacci numbers in multiples
Samacheer Kalvi 8th Maths Guide Chapter 7 Information Processing InText Questions 45
From the above table, we get a general rule as Every kth Fibonacci number is a multiple of F(k).
Answer:
Table 2.
Samacheer Kalvi 8th Maths Guide Chapter 7 Information Processing InText Questions 46
R – Red
Y – Yellow
B – Blue
G – Green

Samacheer Kalvi 8th Maths Guide Chapter 7 Information processing InText Questions

Activity (Text Book Page No. 258)

Code 1: Pigpen

I. Fill in the blank boxes and decode
The Pigpen code looks like meaningless writing, but it is quite easy to catch on to. Each letter is represented by the part of the “Pigpen” that surrounds it.
The first code uses the following key. To complete the code, you need to work out how to use the key to decode the message.
Samacheer Kalvi 8th Maths Guide Chapter 7 Information Processing InText Questions 47

Samacheer Kalvi 8th Maths Guide Chapter 7 Information Processing InText Questions 48
Answer:
Samacheer Kalvi 8th Maths Guide Chapter 7 Information Processing InText Questions 49
To decode
Samacheer Kalvi 8th Maths Guide Chapter 7 Information Processing InText Questions 50

Samacheer Kalvi 8th Maths Guide Chapter 7 Information processing InText Questions

Code 2: Polybius Square Cipher

II. Fill in the blanks
A Polybius Square is a table that allows someone to convert letters into numbers. Use the Polybius square rows and column values to find the code
Samacheer Kalvi 8th Maths Guide Chapter 7 Information Processing InText Questions 51

Samacheer Kalvi 8th Maths Guide Chapter 7 Information Processing InText Questions 52
Answer:
Using this a set of numbers can be used to denote a letter and vise versa
(4, 2) represent T, (3, 4) represent H, (5, 5) represent E
Likewise, when we solve for the remaining. we get
THE NAME OF TREASURE DOES NOT HAVE BAND D .
Since clue 2 gives that the treasure item does not have the letters b & d, therefore “exam pad” & “geometry box” cannot be the treasure item.
Hence, the treasure item is “gift voucher”

Samacheer Kalvi 8th Maths Guide Chapter 7 Information processing InText Questions

Code 3: Atbash Cipher
III. Find the code using the key as shown in given figure:
Atbash cipher is a substitution cipher with just one specific key where all the letters are reversed that is A to Z and Z to A.

Samacheer Kalvi 8th Maths Guide Chapter 7 Information Processing InText Questions 54
Answer:
The given Cipher is:
GSV ILLN MFNYVI RH Z NFOGRKOV LU ULFI ZMW HVEVM
Decoded Message:
THE ROOM NUMBER IS A MULTIPLE OF FOUR AND SEVEN
Since room number is a multiple of 4 & 7,
The room number is 4 × 7 = 28.

Samacheer Kalvi 8th Maths Guide Chapter 7 Information processing InText Questions

Code 4: Using a Key – Reflection Table
IV Use the reflection table which is given below and find the correct word by using a reflected alphabet.
Samacheer Kalvi 8th Maths Guide Chapter 7 Information Processing InText Questions 55
J V A Q B J – _______
P B Z C H G R E G N O Y R – _______
P U N V E – ________
P H O O B N E Q – ________
Answer:
J V A Q B J – WINDOW
P B Z C H G R E G N O Y R – COMPUTER TABLE
P U N V E – CHAIR
P H O O B N E Q – CUPBOARD

After finding the codes, the teacher then asks students to rearrange the clues one by one
CLUES
1. ————————————————-
2. ————————————————-
3. ————————————————-
4. ————————————————-
Answer:
The 4 clues are:
1. Water bottle, Gift voucher, Exam pad, Geometry box
2. The name of the treasure does not have b’ & ‘d’
3. The room is a multiple of four and seven – 28
4. Window, Computer table, Chair, Cupboard

RESULT
(i) The room in which the treasure took place = ________ .
(ii) The place of the treasure = ________ .
(iii) The identity of the treasure = ________ .
(Hint:- If you answered the question number 6 in Exercise 4.3, you can compare and verify your results) The gift voucher contains (20 full marks awarded).
Answer:
The room in which the treasure took place = 28
The place of treasure = Chair
Identity of treasure = Gift voucher.

Samacheer Kalvi 8th Maths Guide Chapter 7 Information processing InText Questions

Try These (Text Book Page No. 260)

Question 1.
Use Pigpen Cipher code and write the code for your name _______ and your chapter names
(i) LIFE MATHEMATICS
(ii) ALGEBRA
(iii) GEOMETRY
(iv) INFORMATION PROCESSING
Answer:
For Pigpen, we should first construct the Criss Cross pattern & then fill with alphabets.
Step 1:
Samacheer Kalvi 8th Maths Guide Chapter 7 Information Processing InText Questions 56

Step 2: Fill the above with all the alphabets from A to Z.
Samacheer Kalvi 8th Maths Guide Chapter 7 Information Processing InText Questions 57

Step 3: So, the secret code is got by the outline of each letter in the above patterns.
Samacheer Kalvi 8th Maths Guide Chapter 7 Information Processing InText Questions 58
similarly for all the others. Therefore pigpen code is
Samacheer Kalvi 8th Maths Guide Chapter 7 Information Processing InText Questions 60

Let us now write the given words with Pigpen code.
Samacheer Kalvi 8th Maths Guide Chapter 7 Information Processing InText Questions 61

Samacheer Kalvi 8th Maths Guide Chapter 7 Information processing InText Questions

Question 2.
Decode the following Shifting and Substitutmg secret codes given below. Which one is easier for you?
(i) Shifting method:- M N S G H M F HR H L O N R R H A K D
Samacheer Kalvi 8th Maths Guide Chapter 7 Information Processing InText Questions 62
Answer:
Decode using shifting method. Let us shift & see by trial & error method.
Let us substitute ‘b’ for ‘a’, ‘e’ for ‘b’ & so on
Samacheer Kalvi 8th Maths Guide Chapter 7 Information Processing InText Questions 63
Message : M N S G H M F H R H L O N R R H A K D
Decoded text : NOTHING IS IMPOSSIBLE
Samacheer Kalvi 8th Maths Guide Chapter 7 Information Processing InText Questions 64
We find that Pigpen code has been used, by substituting for the symbols from the table that we constructed earlier, we go the message.
NOTHING IS IMPOSSIBLE

Samacheer Kalvi 8th Maths Guide Chapter 7 Information processing InText Questions

Question 3.
Write a short message to a friend and get them to decipher it.(Use any one shifting and substituting code method)
Answer:
Student can choose and tr on their own.

Activity (Text Book Page No. 264)

Consider that you are going to a store with your total budget of ₹ 220 to buy things without changing the quantity of the items given in the list below with the following conditions.
Samacheer Kalvi 8th Maths Guide Chapter 7 Information Processing InText Questions 65
Conditions:
(i) First you have to complete the price list given.
(ii) You have to buy three items as per the given price list but within your budget ₹ 220.
(iii) You wont carry exceeding 5kg because you have to walk home carrying them, so they cannot be bulky.

Now, answer the following questions:
1. In how many ways can you buy your items? Complete the price lists given below. One is done for you.
2. Which one is the best purchase price list and why?

Samacheer Kalvi 8th Maths Guide Chapter 7 Information Processing InText Questions 66
Answer:
Samacheer Kalvi 8th Maths Guide Chapter 7 Information Processing InText Questions 67

Samacheer Kalvi 8th Maths Guide Chapter 7 Information processing InText Questions

Samacheer Kalvi 8th Maths Guide Chapter 7 Information Processing InText Questions 68
Answer:
Samacheer Kalvi 8th Maths Guide Chapter 7 Information Processing InText Questions 69

Samacheer Kalvi 8th Maths Guide Chapter 7 Information Processing InText Questions 70
Answer:
Samacheer Kalvi 8th Maths Guide Chapter 7 Information Processing InText Questions 71

Samacheer Kalvi 8th Maths Guide Chapter 7 Information processing InText Questions

Samacheer Kalvi 8th Maths Guide Chapter 7 Information Processing InText Questions 72
Answer:
Samacheer Kalvi 8th Maths Guide Chapter 7 Information Processing InText Questions 73

Samacheer Kalvi 8th Maths Guide Chapter 7 Information processing InText Questions

Samacheer Kalvi 8th Maths Guide Chapter 7 Information Processing InText Questions 74
Answer:
Samacheer Kalvi 8th Maths Guide Chapter 7 Information Processing InText Questions 75

Samacheer Kalvi 8th Maths Guide Chapter 7 Information processing InText Questions

Activity (Text Book Page No. 265)

Consider that you want to buy 12 litres of the same quality of edible oil at your budget price of ₹ 250 per litre. In a supermarket, there are a lot of offers on various oil brands. Some of the offers are given below. Complete the table and find which one is the best offer for you and how much you will save for your total purchase.
Samacheer Kalvi 8th Maths Guide Chapter 7 Information Processing InText Questions 76
Best offer price for you ___________
Amount that you saved for your total purchase ___________
Answer:
Best offer price for you ₹ 137.50
Amount that you saved for your total purchase ₹ 1350

Samacheer Kalvi 8th Maths Guide Chapter 7 Information processing InText Questions

Try These (Text Book Page No. 266)

The teacher divides the class into four groups and setup a mock market in the class room and ask the students to involve in role play as Iwo groups of businessmen and two groups of consumers. Consumers have to buy products at different shops and prepare a price list.

The Iwo supermarkets iii which the two groups buy are Star Food Mart and Super Provisions. Th is week they each have got a special deal on some products. At Star Food Mart. you can buy items at discount prices. At Super Provisions, there are some “BUY ONE GET ONE” deals. Have a look at their deal:
Samacheer Kalvi 8th Maths Guide Chapter 7 Information Processing InText Questions 77

Samacheer Kalvi 8th Maths Guide Chapter 7 Information Processing InText Questions 78

Now answer the following questions.
I. Here is your shopping list:
4 bottles of Protein Milk (200 ml size) 2 packets of Peanut candies(200 gm), 1 packet of Chocolate biscuits, 1 packet of Badam nuts (500 gm)
Answer:
price of Star Food Mart
Samacheer Kalvi 8th Maths Guide Chapter 7 Information Processing InText Questions 80

(i) If you buy all the items in one shop, where will you get the best price?
Answer:
In star Food Mart

(ii) If you buy the items from the two shops, how will you do it to spend the least amount of money?
Answer:
Badam nuts from super provisions and other items from Star Food Mart.

Samacheer Kalvi 8th Maths Guide Chapter 7 Information processing InText Questions

II. You have H 000/- to spend to buy the following shopping list:
6 bottles of Protein Milk (200 ml size), 3 packets of Peanut candies(200 gm), 3 packets of Chocolate biscuits, 1 packet of Badam nuts (250 gm).
Answer:
Comparing two stores
Samacheer Kalvi 8th Maths Guide Chapter 7 Information Processing InText Questions 81

(i) How can you do this so that you dont go over your budget amount ₹ 1000?
Answer:
We can buy them from any one shop.

(ii) Which shop offers you the best value for money on each item?
Answer:
Star Food Mart

Samacheer Kalvi 8th Maths Guide Chapter 7 Information processing InText Questions

(iii) Is the “BUY ONE GET ONE” deal at Super Provisions the same as “50% off” deal?
Answer:
Yes

Samacheer Kalvi 8th Social Science Guide History Chapter 5 Educational Development in India

Tamilnadu State Board New Syllabus Samacheer Kalvi 8th Social Science Guide Pdf History Chapter 5 Educational Development in India Text Book Back Questions and Answers, Important Questions, Notes.

Tamilnadu Samacheer Kalvi 8th Social Science Solutions History Chapter 5 Educational Development in India

Samacheer Kalvi 8th Social Science Educational Development in India Text Book Back Questions and Answers

I. Choose the correct answer.
1. The word ‘Veda’ is derived from …………… .
a) Sanskrit
b) Latin
c) Prakrit
d) Pali
Answer:
a) Sanskrit

2. Which of the following was an important centre for the learning in the ancient period?
a) Gurukula
b) Viharas
c) Pali
d) All of these
Answer:
d) All of these

3. Nalanda, the oldest university in India was located in …………… .
a) Uttar Pradesh
b) Maharashtra
c) Bihar
d) Punjab
Answer:
c) Bihar

4. When did the UNESCO declare Takshashila as world heritage site?
a) 1970
b) 1975
c) 1980
d) 1985
Answer:
c) 1980

5. Which European country were the first to start Modern System of Education in India?
a) British
b) Danish
c) French
d) Portuguese
Answer:
d) Portuguese

6. Which of the following Charter Act made a provision for an annual grant one lakhs Rupees for the promotion of Education in India?
a) Charter Act of 1813
b) Charter Act of 1833
c) Charter Act of 1853
d) Charter Act of 1858
Answer:
a) Charter Act of 1813

7. Which of the following Commission recommended to constitute the University Grants Commission?
a) Sergeant Report, 1944
b) Radhakrishnan Commission, 1948
c) Kothari Commission, 1964
d) National Education Policy, 1968
Answer:
b) Radhakrishnan Commission, 1948

8. In which year the New Education Policy was introduced in India?
a) 1992
b) 2009
c) 1986
d) 1968
Answer:
c) 1986

II. Fill in blanks.
1. The word ‘Veda’ means …………… .
Answer:
Knowledge

2. Taxila ruins were discovered by …………… .
Answer:
Alexander Cunningham

3. …………… was the first ruler to establish a madrasa at Delhi.
Answer:
lltutmish

4. The New Education Policy was revised in …………… .
Answer:
1992

5. …………… is the primary vehicle for implementing the provisions of the Right to Education Act of (RTE) 2009.
Ans: Sarva Shiksha Abhiyan (SSA)

6. Mid-day meal program was introduced in schools in
Answer:
1956

III. Match the following.

1. I – Tsing a. Saraswathi mahal
2. Francis Xavier b. Magna carta of Indian Education
3. Wood’s Despatch c. Western Education in Madras
4. Sarafoji II d. University at Kochin
5. Sir Thomas Munro e. Chinese scholar

Answer:

1. I – Tsing e. Chinese scholar
2. Francis Xavier d. University at Kochin
3. Wood’s Despatch b. Magna carta of Indian Education
4. Sarafoji II a. Saraswathi mahal
5. Sir Thomas Munro c. Western Education in Madras

IV. State True or False.
1. The writings of Charaka and Sushrutha were the sources of learning of medicine.
Answer:
True

2. Temples were the centers of learning and played an active role in the promotion of knowledge.
Answer:
True

3. The Jataka tales tell us that the kings and society took an active interest in promoting education.
Answer:
True

4. Women education in India was not prevalent during the medieval period.
Answer:
False

5. The RMS A scheme was implemented during tenth Five Year Plan.
Answer:
False

V. Consider the following statements and tick the appropriate answer.
1. i) The Nalanda University was founded in fifth century C.E
ii) In ancient India teachers had complete autonomy in all aspects from selection of students to designing their syllabi
iii) In ancient times the teacher was called Kanakkayar. iv) The famous college during the Chola period was Kandhalur salai.
a) i and ii are correct
b) ii and iv are correct
c) iii and iv are correct
d) i, ii and iii are correct
Answer:
d) i, ii and iii are correct

2. Find out the Correct Pair
a) Maktabs – Secondary School
b) Macaulay’s Minutes of 1835 – English education
c) Operation Blackboard – Secondary Education Commission
d) Salabogam – Lands were given to temples
Answer:
b) Macaulay’s Minutes of 1835 – English education

VI. Answer the following in one or two sentences.
1. Write about the importance of Gurukulas.
Answer:

  1. Teaching was oral and students remembered and meditated upon what was taught in the Gurukulas Guru/Acharya.
  2. Many of these Gurukulas were named after the sages. Situated in forests, in service and peaceful surroundings, hundreds of students used to learn together in Gurukulas.
  3. This was known as the ‘Gurukula System’ of education.

2. Name the most notable universities that evolved in ancient India?
Answer:
The most notable universities are Taxila and Nalanda.

3. Write a short note on Taxila?
Answer:

  1. Taxila was an ancient Indian city, which is now in north-western Pakistan.
  2. It is an important archaeological site and the UNESCO declared it as a world heritage site in 1980.
  3. Its fame rested on the university where Chanakya is said to have composed his Arthashastra.

4. Mention the education centres flourished in the Cholas period?
Answer:
Rajaraja Chaturvedi Mangalam was the famous seat of a Vedic college Tirubuvanai Vedic college Tiruvidaikkalai Library and Thiruvaduthurai a medical school

5. Expand SSA and RMSA.
Answer:

  1. Sarva Shiksha Abhiyan (SSA)
  2. Rashtriya Madhyamik Shiksha Abhiyan (RMSA)

6. What do you know about RTE.
Answer:
RTE (Right To Education) provides for free and compulsory education to all children from the age of 6 to 14 years.

VII. Answer the following.
1. What were the sources of education in ancient India?
Answer:
Education in Ancient India:

  1. The historical Sources provide the information that from very early times, the tradition of teaching and learning had been in vogue in India.
  2. The concept of Education might have originated from the Vedas.
  3. The literal meaning of the Sanskrit word ‘Veda’ is knowledge and the word derived from the word Vid, which means ‘to know’.

Gurukula system in ancient India:

  1. In ancient India, both formal and informal education existed.
  2. There were people in homes, villages, and temples who guided young children in imbibing pious ways of life.
  3. Teaching was oral and students remembered and meditated upon what was taught in the Gurukulas Guru / Acharya.

2. Write a paragraph about education under British rule?
Answer:
Education in British rule can be divided into four periods:

  1. Before 1813: Charter of Act of 1813 grant a sum of 1 lakh rupees for the promotion of Indian education.
  2. 1813 to 1853: Macaulay’s minutes of 1835. Higher education was de-orientalized encouraging English education for the upper classes.
  3. 1854 to 1920: The Third phase of British influenced education may be called the period of an All India Educational Policy. It commenced with Sir Charles Wood’s Despatch in 1854.
  4. 1921 to 1947: The fourth phase may be called the period of provincial autonomy.

3. Describe the National Policy on Education.
Answer:
1. The First National Educational Policy of 1968, marked a significant step in the history of education in post-independent India.

2. It aimed to promote national progress, culture, and to strengthen national integration.

3. In 1986, the Government of India introduced a New Education Policy.

4. The aim of New Education Policy (NEP) was to transfer a static society into a vibrant one with a commitment to development and change. It emphasized equal opportunities for marginalized – sections of the country and the removal of disparity through scholarships, adult education, and open universities, especially for rural India.

5. The New Education Policy called for a child-centered approach in primary education.

6. This policy launched the operation of Blackboard to improve primary schools nationwide.

7. The New Education Policy has revised again in 1992.

8. It envisaged the formulation of the National Curriculum Framework, emphasis on in-service education, improvement of facilities, and streamlining of the evaluation system at the secondary stage.

4. Give a detailed account of education under Cholas. Education under Cholas:
Answer:

  • The Chola period was the most brilliant and creative period in Tamil literature.
  • Tamil education enjoyed a greater connection with religion and temple.
  • Inscription denotes the salary of a teacher, method of teaching, food provided to the students, and land given to the schools.
  • Rajaraja Chaturvedi Mangalam was the famous seat of a Vedic college.
  • Tiruvaduthurai inscription of Virarajendra refers to a medical school of this period.

VIII. HOTS
1. How does the flagship programme of SSA achieve Universal Elementary Education?
Answer:
The Sarva Shiksha Abhiyan (SSA) is the Government of India’s flagship programme that was launched in 2000 – 01 to achieve Universal Elementary Education (UEE). SSA is now the primary vehicle for implementing the provisions of the Right of Children to Free and Compulsory Education Act (2009) (RTE). Right To Education (RTE) provides for free and compulsory education to all children from the age of 6 to 14 years. The SSA initiates a variety of innovations and activities related to schools.

Samacheer Kalvi 8th Social Science Educational Development in India Additional Important Questions and Answers

I. Choose the Correct answer.
1. The arrival of the English East India company in the year ……………….
a. 1600
b. 1610
c. 1620
d. 1630
Answer:
a) 1600

2 Elphinstone college located in ……………….
a. Delhi
b. Mumbai
c. Goa
d. Kolkatta
Answer:
b) Mumbai

3. Wardha scheme of Education evolved by ……………… in 1937
a. Gandhiji
b. Nehru
c. Ambedkar
d. Dr.Radha Krishnan.
Answer:
a) Gandhiji

4. Dr.Middleton, started a missionary college at ……………….
a. Calcutta
b. Delhi
c. Chennai
d. Mumbai
Answer:
a) Calcutta

5. In schools all subjects were taught in Tamil from ……………….
a. 1937
b. 1938
c. 1939
d. 1940
Answer:
b) 1938

II. Fill in blanks.
1. The system of Students at an advanced stage of learning guided younger studied younger students known as ……………….
Answer:
Peer learning

2. The family of Guru acts as the ……………… or ……………….
Answer:
School, Ashram

3. Nalanda university located in ………………
Answer:
Rajgir

4. The Muslim Primary schools were called as ……………….
Answer:
Maktabs

5. Secondary schools, under Muslim rule were called as ……………….
Answer:
Madrasas

6. Iltutmish was the fist ruler to establish a madrasa at ……………….
Answer:
Delhi

7. Shahjahanabad Madrasa started by ………………
Answer:
Maulana

8. An education commission under the chairmanship of ……………… was appointed by the Government of India.
Answer:
Dr.S. Kothari

9. Francis Xavier, a ……………… started a university at Cochin.
Ans: Jesuit

10. The educational institution in Kanchi called……………….
Answer:
Ghatika

11. The Vedic College of the Chola period located at ……………….
Answer:
Ennayiram

12. …………… was established during the Nayak rule.
Answer:
Thinnappalli koodam

13. ……………. Chola inscription mentions a library.
Answer:
Tiruvidaikkalai

14. Tiruvidaikkalai inscription belongs to ……………… period.
Answer:
Virarajendra Chola

15. ……………… established the primary schools in Madurai during in Veerappa Nayak period.
Answer:
Fernandez

III. Match the following.

1. The wood’s Despatch a. 1968
2. Education Commission b.1975
3. First National Education policy c.1854
4. Gandhi gram,Rural college d.1956
5. Midday Meal Programme e.1948

Answer:

1. The wood’s Despatch a. 1854
2. Education Commission b.1948
3. First National Education policy c.1968
4. Gandhi gram Rural college d.1975
5. Midday Meal Programme e.1956

IV. State True or False.
1. Equable learning Programme is Samaria Siksha.
Answer:
True

2. Sir. Thomas Munro had a printing press with Devanagari Type
Answer:
False

3. The new Nalanda University is not envisaged as a centre of the inter-civilizational dialogue.
Answer:
False

4. In Gurukula system, the home of Guru used as the school.
Answer:
True

5. The Madras university was founded in 1854.
Answer:
False

V. Consider the following statements and tick the appropriate answer.
1. Choose the correct statement:
1. History of education in British rule can be divided into 4 periods.
a. From the early days of the British rule upto 1813
b. Period from 1813-1853
c. Period from 1854-1920
d. Period from 1921-1947
a. i and iii correct
b. ii and iv correct
c. ii incorrect
d. All are correct
Answer:
d) AH are correct

2. Choose the correct pair:
1. 1834 – Western type education
2. 1855 – Woods Despatch
3. 1856 – Chennai university
4. 1882 – Local Boards Act
Answer:
4.1882 – Wood’s despatch

VI. Answer the following in one or two sentences.
1. Write short notes at Nalanda University.
Answer:

  • The ancient Naianda University was a centre of learning from the 5th century AD (CE) to 12th century AD (CE). Located in present-day Rajgir, Bihar.
  • Nalanda was one of the oldest universities of the world and UNESCO declared the runis of Nalanda Mahavihara a world heritage site.

2. Explain education development in the medieval period.
Answer:

  • The medieval period saw the founding of many religious mutt or monasteries which also took up the cause of education.
  • The Ahobila mutt in Srirangam was one among them where is Sri Ramanuja has made distinctive
    contribution to the cause of education.
  • Besides mutts, Jain pallis and Buddhist vihars played a vital role in educating people where ever the existed. They had large libraries of books in all branches of learning.

3. Write the short notes an Wardha scheme of Education.
Answer:

  • In 1937,Gandhiji evolved a scheme popularly known as the Wardha Scheme of Basic National Education.
  • His system of Education wanted to root out exploitation and centralization in society and create a non-violent social order.

4. Explain the significant development of education in Pallava.
Answer:

  • A significant development took place in the field of education during the Pallava period.
  • Ghatika was an educational institution.
  • The Pallava kings supported those educational institutions through endowments.
  • The Vaishnava ans Saiva mutts provided boarding and lodging facilities to all students and teachers.
  • Kanchi was considered as the main centre of learning.

5. Explain the service of Sir Thomas Munro in education.
Answer:

  • Sir Thomas Munro the Governor of Madras presidency (1820-27) was highly responsible for the introduction of Western education in Madras presidency.
  • He appointed a committee to conduct a statistical survey of the condition of education.

VII. Answer the following.
1. Explain about Gurukula system?
Answer:

  • In ancient India, both formal and informal education existed. Indigenous education was imparted at home, in temples, patashalas, and gurukulas.
  • There were people in homes, villages, and temples who guided young children in imbibing pious ways of life. Temples played a vital role in imparting education and served the centres of learning.
  • Students went to viharas and universities for higher studies. Teaching was oral and students remembered and meditated upon what was taught in the Gurukulas. Many of these Gurukulas were named after die sages.
  • Situated in forests, in serene and peaceful surroundings, hundreds of students used to learn together in Gurukulas.
  • During the early period education was imparted by the teacher (Guru/Acharya) to the pupils who gathered around him and came to live in his house as members of his family. This was known as the Gurukula system of education.

2. Explain the education in Tamil Nadu since independence.
Answer:

  • Free education at the secondary school level was introduced in 1964-65. The Gandhigram Rural College was established in 1975. Since 1971, Distance education has also been introduced in Tamilnadu to educate those who could not go to colleges.
  • In 1956, Midday Meal Programme was introduced in schools. Later, it was extended as a Nutrition Meal Scheme in 1982 to avoid drop-outs in schools.
  • Since 1986 several changes have taken place in the society, in keeping with the National Policy of Education.

Samacheer Kalvi 8th Social Science Guide History Chapter 4 People’s Revolt

Tamilnadu State Board New Syllabus Samacheer Kalvi 8th Social Science Guide Pdf History Chapter 4 People’s Revolt Text Book Back Questions and Answers, Important Questions, Notes.

Tamilnadu Samacheer Kalvi 8th Social Science Solutions History Chapter 4 People’s Revolt

Samacheer Kalvi 8th Social Science People’s Revolt Text Book Back Questions and Answers

I. Choose the correct answer:
1. The Palayakkarar system was instituted in
a) 1519
b) 1520
c) 1529
d) 1530
Answer:
c) 1529

2. Which of the following Palayakkarar of Tamil Nadu was the pioneer against the English rule
a) Pulithevar
b) Yusuf Khan
c) Kattabomman
d) Marudhu brothers
Answer:
a) Pulithevar

3. Colin Jackson was the collector of
a) Madurai
b) Tirunelveli
c) Ramanathapuram
d) Tuticorin
Answer:
c) Ramanathapuram

4. Veera Pandiya Kattabomman was hanged at
a) Panchalamkurichi
b) Sivagangai
c) Tiruppathur
d) Kayathar
Answer:
d) Kayathar

5. Velu Nachiyar was a queen of
a) Nagalapuram
b) Sivagiri
c) Sivagangai
d) Virupachi
Answer:
c) Sivagangai

6. Tiruchirapalli proclamation was issued by
a) Marudhu Pandiyars
b) Krishnappa Nayak
c) Velu Nachiyar
d) Dheeran Chinnamalai
Answer:
a) Marudhu Pandiyars

7. Which of the following place was associated with Dheeran Chinnamalai?
a) Dindigul
b) Nagalapuram
c) Pudukottai
d) Odanilai
Answer:
d) Odanilai

8. Rani Lakshmi Bai led the revolt at
a) Central India
b) Kanpur
c) Delhi
d) Bareilly
Answer:
a) Central India

II. Fill in the Black.
1. The Eastern Palayams were ruled under the control of ………………
Answer:
Kattabomman

2. Vishwanatha Nayakar instituted the Palayakarar system with the consultation of his minister ………………
Answer:
Ariyanatha Mudaliyar

3. The ancestors of Kattabomman belonged to ………………
Answer:
Andhra

4. ……………… was known by Tamils as Veeramangai and Jhansi Rani of south India.
Answer:
Velu Nachiyar

5. ……………… was called as ‘Lion of Sivagangai’.
Answer:
Periya Maruthu

6. ……………… was described the revolt of 1857 as the First War of India Independence.
Answer:
V.D. Savarkar

III. Match the following.

1. Delhi a) Kunwar Singh
2. Kanpur b) Khan Bahadur Khan
3. Jhansi c) Nana Saheb
4. Bareilly d) Lakshmi Bai
5. Bihar e) Bahadur Shah II

Answer:

1. Delhi e) Bahadur Shah II
2. Kanpur c) Nana Saheb
3. Jhansi d) Lakshmi Bai
4. Bareilly b) Khan Bahadur Khan
5. Bihar a) Kunwar Singh

IV. State true or false.
1. The Vijayanagar rulers appointed Nayaks in their provinces.
Answer:
True

2. Sivasubramania was the minister of Marudhu Pandiyas.
Answer:
False

3. Kattabomman was hanged on 17th October 1799.
Answer:
True

4. Fettah Hyder was the elder son of Tipu Sultan.
Answer:
True

V. Consider the following statements and tick(✓) the appropriate answer.
i) The Vellore revolt was held in 1801.
ii) The family members of Tipu were imprisoned at Vellore fort after the fourth Mysore war.
iii) At the time of the Vellore revolt, the Governor of Madras was Lord William Gentinck.
iv) The victory of the revolt of Vellore against the British was one of the significant events in the history of India.
a) i & ii are Correct b) ii & iv are Correct
c) ii &iii are correct ‘ d) i, ii & iv are correct
Answer:
c) ii & iii are correct

a) Find out the wrong pair
1. Marudu Pandiyar – a. Ettayapuram
2. Gopala Nayak – b. Dindigul
3. Kerala Varma – c. Malabar
4. Dhoondaji – d. Mysore
Answer:
1. Marudu Pandiyar-a. Ettayapuram

b) Find out the odd one
Kattabomman, Oomaithurai, Sevathaiah, Tipusultan

VI. Answer the following in one or two sentences.
1. What you know about the Palayakarars? Name some of them.
Answer:
1. Palayakkarar was the holder of a territory or a Palayam. These Palayams were held in military tenure and extended their full cooperation to be need of the Nayaks. The Palayakkarars collected taxes, of which one-third was given to the Nayak of Madurai another one-third for the expenditure of the army, and the rest was kept for themselves.

2. Kattabomman, Pulithevan, Marudu Brothers, Dheeran Chinnamalai.

2. What was the part of Velu Nachiyar in the Palayakkarar revolt?
Answer:

  • Velu Nachiyar organised an army and employed her intelligent agents to find where the British stored their ammunition.
  • She recaptured Sivagangai and was again crowned as a queen with the help of the Marudu brothers.

3. Who were the leaders of the Palayakkarar confederacy in the south Indian rebellion?
Answer:
Marudu Pandiar of Sivaganga, Gopala Nayak of Dindugal, Kerala Varma of Malabar and Krishnappa Nayak, and Dhoondaji of Mysore.

4. What was the importance of Tiruchirappalli proclamation?
Answer:

  • The marudu Pandyas issued a proclamation of Independence called Tiruchirappalli Proclamation in June 1801,
  • It was the first call to the Indians to unite against the British.

5. Bring out the effects of the Vellore revolt.
Answer:
Effects of the Vellore Revolt:

  1. The new methods and uniform regulations were withdrawn.
  2. The family of Tippu as a precautionary measure was sent to Calcutta.
  3. William Cavendish Bentinck was removed from his service.

6. What was the immediate cause of the Revolt of 1857?
Answer:
The immediate cause was the introduction of new Enfield Riffles in the army. For loading it, we want to use our mouth to remove the cartridge of the rifle.The cartridges were greased by the fat of cow and the pig.

VII. Answer the following in detail.
1. What do you know about the Pulithevar?
Answer:
1. Pulithevar was the pioneer in Tamil Nadu, to protest against the English rule in India. He was the Palayakkarar of the Nerkattumseval, near Tirunelveli.

2. During his tenure he refused to pay the tribute neither to Mohammed Ali, the Nawab of Arcot nor to the English. Further, he started opposing them. Hence, the forces of the Nawab of Arcot and the English attacked Pulithevar. But the combined forces were defeated by Pulithevar at Tirunelveli.

3. Pulithevar was the first Indian king to have fought and defeated the British in India. After this victory, Pulithevan attempted to form a league of the Palayakkars to oppose the British and the Nawab.

4. In 1759, Nerkattumseval was attacked by the forces of Nawab of Arcot under the leadership of Yusuf Khan.

5. Pulithevar was defeated at Anthanallur and the Nawabs forces captured Nerkattumsevval in 1761.

6. Pulithevar who lived in exile recaptured Nerkattumseval in 1764. Later, he was defeated by Captain Campell in 1767. Pulitzer escaped and died in exile without fulfilling his purpose, although his courageous trail of a struggle for independence in the history of South India.

2. Explain the events that led to conflict between Dheeran Chinnamalai and the british.

  • Dheeran Chinamalai was trained by French militray in modem welfare.
  • He was along the side Tipu Sultan to fight against the British East India Company and got victories against the British.
  • After Tipu Sulthan’s death, Chinnamalai settled down at Odanilai and constructed a fort there to continue his struggle against the British.
  • He sought the help of Marathas and Maruthu Pandiyar to attack the British at Coimbatore in 1800.
  • Chinnamalai was forced to attack Coimbatore on his own.
  • His army was defeated the British in battles at Cauvery, Odanilai, and Arachalur.
  • During the final battle, Chinnamalai was betrayed by his cook Nallapan and was hanged in Sankagiri Fort in 1805.

3. What were the causes of the Great revolt of 1857?
Answer:
1. The most important cause of revolt in 1857 was a popular discontent of the British policy of economically exploiting India. This hurt all sections of society. The peasants suffered due to high revenue demands and the strict revenue collection policy.

2. Policies of the doctrine of lapse, subsidiary alliance, and policy of Effective Control created discontentment among people. The annexation of Oudh proved that even the groveling loyalty can’t satisfy British greed for territories.

3. The conversion activities of Christian missionaries were looked upon with suspicion and fear. The priests and the maulavis showed their discontent against the British rule.

4. Abolition of practices like sati, female infanticide, support to widow remarriage, and female education was seen by many as interference in their Indian culture by the Europeans.

5. The Indian sepoys were looked upon as inferior beings and treated with contempt by their British officers. They were paid much less than the British soldiers. All avenues of the promotion were closed to them as all the higher army posts were reserved for the British.

4. What were the causes of the failure of the Revolt of 1857?
Causes for the failure of the Revolt of 1857:
Various causes were responsible for the failure of the revolt.

  • Lack of organisation, discipline common plan of action, centralised leadership, modem weapons, and techniques.
  • The rebel leaders were no match to the British Generals. Rani Lakshmi Bai, Tantia Tope, and Nana Saheb were courageous but not good generals.
  • Non-participation of Bengal, Bombay, Madras, Western Punjab, and Rajputana.
  • The modem educated Indians did not support the revolts because the English rule could reform the Indian society and modernize it.
  • The British managed to get the loyalty of the Sikhs, Afghans, and Gurkha regiments who helped in the suppression of the revolt.
  • The British had better weapons, better generals, and good organisation.

VIII. HOTS
1. Prove that there was no common purpose among the leaders of the Great revolt of 1857.
Answer:

  1. The revolt did not have a clear leader. Bahadur Shah II was old and exhausted. He lacked military leadership.
  2. At Kanpur, Nana Sahib was proclaimed a Peshwa, who was the adopted son of the exited Peshwa Baji Rao II.
  3. He believed that he was entitled to a pension from the British.
  4. When he was refused to be given, he was compelled to revolt against them. Rani Lakshmi Bai and Tantia Tope fought for Jhansi and Gwalior.
  5. Begun of Awadh wanted to safeguard her territory.
  6. Khan Bahadur Khan fought against the British when they subjugated his government in Bareilly.
  7. As such, they lacked unity and gave way for the easy victory of the British.

Samacheer Kalvi 8th Social Science People’s Revolt Additional Important Questions and Answers

I. Choose the Correct Answer.
1. The Governor ordered Kattabomman to surrender
a) Canning
b) Robert Clive
c) Edward Clive
d) Correnwalis
Answer:
c) Edward Clive

2. Dheeran Chinnamalai was trained by the military is the modern welfare
a) Netherland
b) British
c) Germany
d) French
Anwer:
French

3. Puli Thevar was defeated by ……………. in 1767
a) Yusuf Khan
b) Jackson
c) Arcot Newab
d) Captain Campbell
Answer:
d) Captain Campbell

4. After the 1857 Revolt, the Governor-General was called as ……………..
a) Viceroy
b) Major
c) Carnal
d) Sir
Answer:
a) Viceroy

5. In 1858, the proclamation released by Queen ……………..
a) Elizabeth
b) Victoria
c) Mary Ann
d) Isabellah

Answer:
b) Victoria

6. Delhi was recaptured by ………………… in 1857 Revalt
a) Nicholson
b) Gilesphey
c) Camphell
d) Canning
Answer:
a) Nicholson

7. The Mughal emperor ………………… as the symbol of the Revolt 1857
a) Akbar
b) Shajahan
c) Aurangazeb
d) Bahadur shah II
Answer:
d) Bahadur shah II

8. Vellore Revolt held in the year ………………….
a) 1806
b) 1805
c) 1801
d) 1800
Answer:
a) 1806

9. The ………………… rulers appointed Nayak in their Provinces.
a) Chola
b) Pallava
c) Pandya
d) Vijayanagara
Answer:
d) Vijayanagara

10. Dheeran Chinnamalai fought against the ………………… East India company
a) French
b) British
c) Dutch
d) Denish
Answer:
b) British

II. Fill in the Blanks.
1. ………………… was the pioneer for protest against the English role of India.
Answer:
Puli Thevar

2. The whole country was divided into ………………… palayams under the Palayakkarar system.
Answer:
72

3. ………………… asked Kattabomman to pay the tribute arrears.
Answer:
Coiin Jackson

4. The minister of Kattabomman was ………………….
Answer:
Siva subramanian

5. The Tiruchirappali proclamation released in the year ………………….
Answer:
1801

6. ………………… also known as Jansi Rani of South India.
Answer:
Velunatchiyar

7. ………………… died in 1772 Sivaganga battle.
Answer:
Muthu Vaduga Nathar

8. The Thiruchirappalli Proclamation was placed on the walls of the ………………… temple.
Answer:
Srirangam vaishnava Temple

9. ………………… System was abolished by the Treaty of kamataka in 1801
Answer:
Palayakkarar

10. Military General Agnew introduced a new turban known as ………………….
Answer:
‘Agnew’s turban

11. The immediate cause of sepoy revolt was the introduction of ………………….
Answer:
Enfield Riffles

12. Due to the terrorist activities of Chinna maruthu against British called as ………………….
Answer:
Lion of Sivaganga

13. The Revolt of 1857 headed by ………………… in Kanpur.
Answer:
Nana Saheb

14. The Revolt in Delhi controlled by the British General ………………….
Answer:
John Nicholson

15. The sepoy Revolt was led by ………………… in Gwalior.
Answer:
Tantia Tope

III. Match the following.

1. Canning a) Commander
2. Kuyili b) Governor
3. Edword Clive c) Colonel
4. Colin Jackson d) Govemer General
5. Agnew e) Collector

Answer:

1. Canning d) Gevemor General
2. Kuyili a) Commander
3. Edword Clive b) Gevemor
4. Colin Jackson e) Collector
5. Agnew c) Colonel

IV. State True or False.
1. Dheeran Chinnamalai was bom at Melapalayam in Chennimalai near Erode
Answer:
True

2. The Maratha brothers were executed in the port of Tirupatur in Ramanathapuram District on 10th October 1801.
Answer:
False

3. The Palayakkarar of Sivagiri was not a tributory to the company.
Answer:
False

4. The British military policy came to be dominated by the idea of’divide and counterpoise’
Answer:
True

V. Considder the following statements and tick (✓) the appropriate answer.
1. Choose the correct statement
1) Velu Nachiyar was a queen of Sivagangai.
2) At the age of 16, She was married to Muthu Vaduganathar, the Raja of Sivagangai.
3) In 1772, the Nawab of Arcot and the British troops invaded Sivagangai
4) They killed Muthu Vaduganathar in the battle of Kalayarkoil.
a) 3 is correct
b) 2 is correct
c) All are correct
d) All are incorrect
Answer:
c) All are correct

a) Find out the wrong pair:
1. Dheeran Chinnamalai – Nallapan
2. 1800-1801 – Second Palayakkarar war
3. Velu Nachiyar – Kuyili
4. Kattabomman – John Cradock
Answer:
4. Kattabomman – John Cradock

b) Find out the odd one
1. a) Kattabomman
b) Siva Subramanian
c) Vijaya Ragunatha Thondaiman
d) Oomathurai
Answer:
c) Vijaya Ragunatha Thondaiman

VI. Answer the following in one or two sentences.
1. What were the two blocs of Palayakkarars? who ruled those blocs?.
Answer:

  • There were two blocs of Palayams Eastern Palayam and western Palayam.
  • The Palayams were ruled by Poligars.

2. What was the immediate cause of the 1806 Vellore Revolt?.
Answer:

  • In June 1806, military General Agnew introduced a new turban, resembling a European hat with a badge of the cross on it.
  • It was popularly known as ‘Agnew’s turban’. Both the Hindu and Muslim soldiers opposed it
  • So the soldiers were severely punished by the English.
    This was the immediate cause of the 1806 Vellore revolt.

3. Name the places of the 1857 Revolt.
Answer:
Delhi, Lucknow, Kanpur, Jhansi & Gwalior, Bareilly, Bihar.

VII. Answer the following in detail.
1. Explain the consequences of the Revolt of 1857.
Answer:

  • The Revolt of 1857 marked a turning point in the history of India. It led to changes in the system of administration and the policy of the Government.
  • The administration of India was transferred from the East India Company to the British Crown through the ‘Queen’s Proclamation’ in 1858. The governor-general was given the title of viceroy.
  • The Board of Directors and the Board of Control were replaced by the Council of 15 members headed by the Secretary of State to supervise Indian affairs.
  • The Indian Army was thoroughly reorganized. More Britishers were employed in the army.
  • The British military policy came to be dominated by the idea of divide and counterpoise’. In fact, the Revolt of 1857 played an important role in bringing the Indian people together and imparting them the consciousness of belonging to one country.
  • The Revolt paved the way for the rise of the modem national movement century that the 1857 Revolt came to be interpreted as a “planned war of national independence by the Historian V.D. Savarkar in his book, ‘First War of Indian Independence.

Samacheer Kalvi 8th Maths Guide Chapter 7 Information processing Ex 7.4

Tamilnadu State Board New Syllabus Samacheer Kalvi 8th Maths Guide Pdf Chapter 7 Information processing Ex 7.4 Text Book Back Questions and Answers, Notes.

Tamilnadu Samacheer Kalvi 8th Maths Solutions Chapter 7 Information processing Ex 7.4

Question 1.
Find the best buy of the following purchases:
(i) A pack of 5 chocolate bars for ₹ 175 or 3 chocolate bars for ₹ 114?
Answer:
Samacheer Kalvi 8th Maths Guide Answers Chapter 7 Information Processing Ex 7.4 1
Best buy is a packet of 5 chocolate bars for ₹ 175

Samacheer Kalvi 8th Maths Guide Chapter 7 Information processing Ex 7.4

(ii) Basker buy 1\(\frac { 1 }{ 2 }\) dozen of eggs for 81 and Aruna buy 15 eggs for ₹ 64.50?
Answer:
Samacheer Kalvi 8th Maths Guide Answers Chapter 7 Information Processing Ex 7.4 2
Best buy is 15 eggs for ₹ 64.5

Question 2.
Using the given picture find the total special offer price of fresh sweets and bakery products to buy \(\frac { 1 }{ 2 }\) kg laddu, 1 kg cake, 6 pockets of bread.
Samacheer Kalvi 8th Maths Guide Answers Chapter 7 Information Processing Ex 7.4 3
Answer:
Samacheer Kalvi 8th Maths Guide Answers Chapter 7 Information Processing Ex 7.4 4

Samacheer Kalvi 8th Maths Guide Chapter 7 Information processing Ex 7.4

Question 3.
Using the given picture prepare a price list.
Suppose you planto buy 1\(\frac { 1 }{ 2 }\) kg of apple, 2 kg of pomegranate, 2 kg of banana, 3 kg of mango, \(\frac { 1 }{ 2 }\) kg of papaya, 3 kg of onion, \(\frac { 1 }{ 2 }\) kg of tomato, and 1 kg of carrot in shop 1, how much will you save compared to shop 2.
Shop 1
Samacheer Kalvi 8th Maths Guide Answers Chapter 7 Information Processing Ex 7.4 5

Shop 2
Samacheer Kalvi 8th Maths Guide Answers Chapter 7 Information Processing Ex 7.4 6
Answer:
Samacheer Kalvi 8th Maths Guide Answers Chapter 7 Information Processing Ex 7.4 7
Samacheer Kalvi 8th Maths Guide Answers Chapter 7 Information Processing Ex 7.4 8
Price in shop 2 – Shop 1 = 715 – 675.75
= ₹ 39.25
We can save ₹ 39.25 in shop 1 compared to shop 2.

Samacheer Kalvi 8th Maths Guide Chapter 7 Information processing Ex 7.4

Question 4.
You want to buy some grocery items as per your shopping list that are given in the picture with their price. Also you have a bag that capacity of carrying 7 kg. Using weight ratio approach tabulate to find the total price and how much can
you buy more grocery items within your budget of ₹ 1000 and not exceeding 7 kg.
Shopping list
1. 2kg of red chili
2. 2 kg of coriander
3. 1 kg of garlic
4. 1 kg of tamarind
5. 2 kg of toor dal

Red chilli
Samacheer Kalvi 8th Maths Guide Answers Chapter 7 Information Processing Ex 7.4 9
₹ 141/1 kg

Coriander
Samacheer Kalvi 8th Maths Guide Answers Chapter 7 Information Processing Ex 7.4 10
₹ 130/1 kg

Garlic
Samacheer Kalvi 8th Maths Guide Answers Chapter 7 Information Processing Ex 7.4 11
₹ 82/1 kg

Tamarind
Samacheer Kalvi 8th Maths Guide Answers Chapter 7 Information Processing Ex 7.4 12
₹ 99/1 kg

Toor dal
Samacheer Kalvi 8th Maths Guide Answers Chapter 7 Information Processing Ex 7.4 13
₹ 78/1 kg
Answer:
Samacheer Kalvi 8th Maths Guide Answers Chapter 7 Information Processing Ex 7.4 14

Samacheer Kalvi 8th Maths Guide Chapter 7 Information processing Ex 7.4

Objective Type Questions

Question 5.
Online or television advertisements influence people on spending decisions by
(a) using special music
(b) making them think that they need the item
(c) using attractive pictures
(d) all the above
Answer:
(d) all the above

Question 6.
When I go shopping, I will buy
(a) something that looks attractive
(b) something my friend has
(c) something that I need to purchase
(d) the first thing I see in the store
Answer:
(c) something that I need to purchase

Samacheer Kalvi 8th Maths Guide Chapter 7 Information processing Ex 7.4

Question 7.
The best shopping choice is to
(a) shop at brand name stores always buy
(b) compare the choices before buying
(c) the same thing my friends bought
(d) buy at a regular shop always
Answer:
(b) compare the choices before buying

Samacheer Kalvi 8th Social Science Guide Geography Chapter 8 Map Reading

Tamilnadu State Board New Syllabus Samacheer Kalvi 8th Social Science Guide Pdf Geography Chapter 8 Map Reading Text Book Back Questions and Answers, Important Questions, Notes.

Tamilnadu Samacheer Kalvi 8th Social Science Solutions Geography Chapter 8 Map Reading

Samacheer Kalvi 8th Social Science Map Reading Text Book Back Questions and Answers

I. Choose the best answer.
1. The subject which deals with map making process is ……………….. .
a) Demography
b) Cartography
c) Physiography
d) Topography
Answer:
b) Cartography

2. A map that shows the physical features of an area is called ………………. .
a) Cadastral map
b) Relief map
c) Climatic map
d) Resource map
Answer:
a) Cadastral map

3. Shallow water bodies are represented by ……………. colour
a) Yellow
b) Brown
c) Light blue
d) Dark blue
Answer:
c) Light blue

Samacheer Kalvi 9th Social Science Guide Geography Chapter 7 Mapping Skills

4. The maps which are known as plans are
a) Cadastral maps
b) Topographical maps
c) Isoline maps
d) Transport maps
Answer:
a) Cadastral maps

5. Actual distribution of population can be represented by …………….. .
a) lines
b) Shades
c) Dots
d) Contours
Answer:
b) Shades

II. Fill in blanks.
1. The globe is the true representation of the ……………… .
Answer:
Earth

2. A way of representing the spherical earth on a flat surface is …………….. .
Answer:
Map

Samacheer Kalvi 9th Social Science Guide Geography Chapter 7 Mapping Skills

3. A line that joins the points of equal elevation is ……………. .
Answer:
Isolines

4. Cadastral maps are usually maintained by…………….. .
Answer:
Thematic map

5. …………… map is focused on a specific theme.
Answer:
Local administrator

III. Choose the option which matches the following correctly.
1. Legend – 45°
2. North East – brown colour
3. Contour Line – thematic map
4. Cadastral map – key of a map
5. Choropleth – taxation
a) 3,5,1,4,2
b) 4,1,2,5,3
c) 2,5,1,3,4
d) 5,2,4,1,3
Answer:
c) 2,5,1,3,4

IV. Match the statement with the reason and select the correct answer.
1. Statement : Small scale maps can show only major features.
Reason : Due to lack of space ,it shows large areas like Continents and countries,
a) Statement is true but reason is wrong.
b) Statement is wrong and reason is correct,
c) Both the statement and reasons are correct
d) Both the statement and reasons are wrong.
Answer:
c) Both the statement and reasons are correct

Samacheer Kalvi 9th Social Science Guide Geography Chapter 7 Mapping Skills

2. Statement : The conventional signs and symbols are the keys of map reading.
Reason : These symbols give a lots of information in a limited area.
a) Both the statement and reasons are correct.
b) Statement is wrong and reason is correct.
c) Statement is true but reason is wrong.
d) Both the statement and reasons are wrong.
Answer:
a) Both the statement and reasons are correct

V. Answer the following in a sentence or two.
1. Define “Map scale”.
Answer:

  1. Map scale refers to the relationship (or ratio) between distance on a map and the corresponding distance on the ground.
  2. The map scale is stated in words i.e., 1 cm to 1 km.

2. What is a physical map?
Answer:
Map which shows the physical feature of an area is called a physical map.

3. Write a short note on map projection.
Answer:A map projection is a way of representing the spherical earth on a flat surface of a map. The curved surface of the earth cannot be shown accurately on a map. So, cartographers use map projections while mapping the earth surface which would help them to reduce distortions.

Samacheer Kalvi 9th Social Science Guide Geography Chapter 7 Mapping Skills

4. Name the Intermediate directions.
Answer:
The intermediate directions are northeast, north west, south east, and south west.

5. What are the uses of a cadastral map?
Answer:
Cadastral maps are useful for local administration such as the city survey, taxation, management of estates and to define property in legal documents.

VI. Differentiate.
1. Relief map and thematic map.

Relief Map Thematic Map
Map that shows the physical features of an area is Usually called Relief map. Map that focuses on a specific them or subject area.
It shows landforms like rivers, mountains. It shows phenomena like temperature variation, rainfall distribution and population density.

2. Large scale map and small scale map.

Large scale Map Small Scale Map
Large scale maps portray the information in detail than the small scale maps. Small scale maps shows only major feature omitting the minor ones due to lack of space. Large areas like continents or countries small scale maps are used.

3. Globe and Map.

Globe Map
Globe gives a three-dimensional representation of the entire world and it is a miniature form of the earth. Map gives a two-dimensional representation of certain regions or the entire world.

VII. Answer in a paragraph.
1. Explain the different types of scales in detail.
Answer:
Scales on maps can be represented in three different ways. They are:

  1. Statement or Verbal scale
  2. Representative Fraction (RF) or Ratio Scale
  3. Graphical or Bar Scale

1. Statement or Verbal scale:

  • In this method, the map scale is stated in words i.e., 1 cm to 1 km.
  • It means 1 cm distance on the map corresponds to 1 km distance on the ground.
  • Thus it is written on the map like 1 cm to 1 km.

2. Representative Fraction (RF) or Numerical Fraction or Ratio Scale:

  • It shows the relationship between the map distance and the corresponding ground distance in the same units of length.
  • R.F. is generally shown as a fraction.

3. Graphical or Bar Scale or Linear Scale:

  • A graphic scale looks like a small ruler drawn at the bottom of the page.
  • This line is divided and sub-divided into lengths each of which represents a certain distance on the ground.
  • This scale has an added advantage for taking copies of maps as the measurement does not change.

Samacheer Kalvi 9th Social Science Guide Geography Chapter 7 Mapping Skills

2. Describe the Cadastral map and its importance.
Answer:

  • The Cadastral map refers to a map which shows the boundaries and ownership of land within a specified area.
  • They are sometimes called plans.
  • They show full details of boundaries and buildings.
  • They are useful for local administration such as city survey, taxation to define property in legal documents.
  • It is maintained by the government as a public record.

Importance:

  • It surveys documents boundaries of land ownership, diagrams, sketches plan, and maps.
  • They are originally used to ensure reliable facts for land valuation and taxation.

3. Write a paragraph about the conventional signs and symbols.
Answer:
The conventional signs and symbols:

  1. Conventional signs are symbols used in maps to represent different features.
  2. The symbols are explained in the key of the map.
  3. These symbols give a lot of information in a limited space.
  4. With the use of these symbols, maps can be drawn easily and the concept of the map can be understood well. There is an international agreement regarding the use of certain symbols.
  5. The symbols that fall under this category are Called Conventional Symbols. Another category is called contextual symbols which are decided by the cartographers.

Samacheer Kalvi 8th Social Science Map Reading Additional Important Questions and Answers

I. Choose the Correct answer.
1. A choropleth map is a ………………… Map.
a) Physical
b) Thematic
c) Cadastral
d) All of these
Answer:
b) Thematic

2. Each dot on a dot – density map represents some amount of …………………
a) data
b) water
c) equal
d) All of these
Answer:
a) data

Samacheer Kalvi 9th Social Science Guide Geography Chapter 7 Mapping Skills

3. Cadastral is derived from ………………… word.
a) french
b) Latin
c) greek
d) English
Answer:
a) french

4. The ………………… surface of the earth cannot be shown accurately on a map.
a) flat
b) curved
c) spherical
d) None of the above
Answer:
b) curved

5. Latitudes and longitudes marked on the map give the ………………… information of the area.
a) Locational
b) Physical
c) Thematical
d) Directional
Answer:
a) Location

6. The ………………… cannot draw maps the same size as the land.
a) Demography
b) Cartography
c) Physiography
d) Topography
Answer:
b) Cartography

7. A ………………… looks like a small ruler drawn at the bottom of the page.
a) Verbal scale
b) Bar scale
c) Legend
d) Direction
Answer:
b) Bar scale

8. A ………………… is included in a map to unlock it.
a) Verbal scale
b) Bar scale
c) Legend
d) Direction
Answer:
c) Legend

Samacheer Kalvi 9th Social Science Guide Geography Chapter 7 Mapping Skills

9. ………………… is of done by means of an arrow pointing to the north
a) Verbal scale
b) Bar scale
c) Legend
d) Direction
Answer:
d) Direction

10. The ………………… should normally be given outside the frame of the map.
a) Source
b) Bar scale
c) Legend
d) Direction
Answer:
a) Source

II. Fill in the blanks.
1. Each map has its own ………………… which is indicated on the map.
Answer:
Scale

2. ………………… tells about the content of the map.
Answer:
Title

3. ………………… shows the relationship between the map distance and the corresponding ground distance.
Answer:
Representative fraction

4. A ………………… looks like a small ruler drawn at the bottom of the page.
Answer:
graphic scale

Samacheer Kalvi 9th Social Science Guide Geography Chapter 7 Mapping Skills

5. Directions on a map are often given with a symbol called ………………….
Answer:
a Compass rose

6. Maps and globe an important tools for …………………
Answer:
Geographers

7. ………………… is an act of interpreting or understanding the geographic information portrayed on a map.
Answer:
Map reading

8. Globe gives a dimensional Representation of the entire world.
Ans: three

9. ………………… and ………………… marked on maps give the locational information of the area covered in the respective maps.
Answer:
Latitudes, Longitudes

10. ………………… give a lot of information in a limited space.
Answer:
Conventional signs

11. The meaning cadastral, is ………………….
Answer:
Register of Territorial Property

12. ………………… are classified into qualitative and quantitative thematic maps.
Answer:
Thematic maps

Samacheer Kalvi 9th Social Science Guide Geography Chapter 7 Mapping Skills

13. ………………… is an act of interpreting the geographic information Portuguese on a map
Answer:
Map Reading

14. In the represents ………………… distance
Answer:
1 km

15. In the map Legend symbol denotes …………………
Answer:
camp

III. Choose the option which matches the following correctly.

1. Map a) Lines connect the same value
2. Cartography b) Register of Territorial property
3. Map scale c) Mapmaking
4. Cadastre d) two-dimensional representation
5. Isoline e) relationship between distance

Answer:

1. Map a) Lines connect the same value
2. Cartography b) Register of Territorial property
3. Map scale c) Mapmaking
4. Cadastre d) two-dimensional representation
5. Isoline e) relationship between distance

IV. Match the statement with the reason and select the correct answer.
1. Statement: A map projection is a way of representing the spherical earth on a flat surface of a map.
Reason: The curved surface of the earth cannot be shown accurately on a map.
a) Statement is true but the reason is wrong.
b) Statement is wrong and the reason is correct,
c) Both the statement and reasons are correct.
d) Both the statement and reasons are wrong.
Answer:
c) Both the statement and reasons are correct

Samacheer Kalvi 9th Social Science Guide Geography Chapter 7 Mapping Skills

2. Statement: map key or legend is included in a map to unlock it.
Reason: It gives you the information needed for the map to make sense.
a) Statement is true but the reason is wrong.
b) Statement is wrong and the reason is correct.
c) Both the statement and reasons are correct.
d) Both the statement and reasons are wrong.
Answer:
c) Both the statement and reasons are correct

V. Answer the following in a sentence or two.
1. Why Is cadastral important?
Answer:
Codastral map survey documents the boundaries of land ownership, by the production of document diagram, sketches, plans, charts, and maps.

2. What are two kinds of Thematic maps?
Answer:
A thematic map is classified into qualitative and quantitative thematic maps.

Samacheer Kalvi 9th Social Science Guide Geography Chapter 7 Mapping Skills

3. What is a Qualitative map?
Answer:
A qualitative map is in the form of quality and expresses the presence or absence of the subject on a map like the kind of vegetation.

4. What are Isobars?
Answer:
Isobars shows the distribution of atmospheric pressure and isotherms showing the distribution of temperature.

5. State any two uses of maps.

  • To find the location of objects and places.
  • Display weather conditions.

VI. Differentiate.
1. Conventional signs and Symbols.

Conventional signs Symbols
Conventional signs are symbols used in maps to represent different features. The symbols are explained in the key of the map. These symbols give a lot of information in a limited space.

2. Cadastral Map and Thematic Map.

Cadastral Map Thematic Map
A cadastral map refers to a map that shows the boundaries and ownership of land within a specified area. These maps are sometimes known as plans. A thematic map is a map that focuses on a specific theme or subject area such as physical phenomena like temperature variation, rainfall distribution and population density in an area.
As they are on large scale, they show full details of the boundaries and buildings. Thematic maps emphasize spatial variation of human issues like population density or prevalence of diseases.

VII. Answer in a paragraph.
1. What are the uses of maps?
Answer:

  • To find location of objects and places.
  • To find the transportation routes.
  • Maps show strategic location is useful for the military.
  • Serve as a tourist guide.
  • To find the spatial distribution of different phenomena.
  • Display weather conditions.
  • Highly helpful in learning geography.
  • Represent the real world on small scale.

Samacheer Kalvi 9th Social Science Guide Geography Chapter 7 Mapping Skills

2. What are the basic components of a map? Explain any two.
Answer:
The basic components of a map are

  • Title
  • Scale
  • Legend or key
  • Direction
  • Source
  • Map projection and locational information and
  • Conventional signs and symbols.

Title:
The title tells about the content of the map and is placed mostly at the top comer or at the bottom comer of the map.

Legend or key:

  • A map key or Legend is included to unlock the map.
  • It gives us the needed information.
  • Maps often use symbols or colours to represent things and the key explains what they mean.
  • Symbols are the key might be pictures or icons that represent different things on the map.
  • Some examples are map legend.
  • Campground.
  • Emergency Telephone.
  • Gravel Road.

Samacheer Kalvi 8th Social Science Guide Geography Chapter 7 Exploring Continents Africa, Australia and Antarctica

Tamilnadu State Board New Syllabus Samacheer Kalvi 8th Social Science Guide Pdf Geography Chapter 7 Exploring Continents Africa, Australia and Antarctica Text Book Back Questions and Answers, Important Questions, Notes.

Tamilnadu Samacheer Kalvi 8th Social Science Solutions Geography Chapter 7 Exploring Continents Africa, Australia and Antarctica

Samacheer Kalvi 8th Social Science Exploring Continents Africa, Australia and Antarctica Text Book Back Questions and Answers

I. Choose the correct answer.
1. The southernmost tip of Africa is.
a) Cape Blanca
b) Cape Agulhas
c) Cape of Good Hope
d) Cape Town
Answer:
c) Cape of Good Hope

2. The manmade canal through an isthmus between Egypt and Sinai Peninsula is
a) Panama Canal
b) Aswan Canal
c) Suez Canal
d) Albert Canel
Answer:
c) Suez Canal

Samacheer Kalvi 9th Social Science Guide Geography Chapter 7 Mapping Skills

3. In respect of the Mediterranean climate, consider the following statements, and choose the correct answer.
(1) The average rainfall is 15cm
(2) The summers are hot and dry, winters are rainy.
(3) Winters are cool and dry, Summers are hot and wet
(4) Citrus fruits are grown
a) 1 is correct
b) 2 and 4 are correct
c) 3 and 4 are correct
d) All are correct
Answer:
b) 2 and 4 are correct

4. The range which separates the west and east flowing rivers in Australia is
a) Great Dividing Range
b) Himalayan Range
c) Flinders range
d) Mac Donnell Range
Answer:
a) Great Dividing Range

5. Kalgoorile is famous for ………. mining.
a) Diamond
b) Platinum
c) Silver
d) Gold
Answer:
d) Gold

II. Fill in the blanks.
1. Atlas Mountain is located in ………………. continent
Answer:
African

2. ………………. is the highest peak of Africa.
Answer:
Mt. Kilimanjaro

Samacheer Kalvi 9th Social Science Guide Geography Chapter 7 Mapping Skills

3. ………………. is the most common tree in Australia.
Answer:
Eucalyptus

4. A temperate grass land of Australia is called ………………. .
Answer:
Downs

5. ………………. is the first Indian research station in Antarctica.
Answer:
Dakshi Gangotri

III. Match the following.

1. Pinnacle a) Equatorial forest.
2. Krill b) salt lake
3. Ostrich c) small red fish
4. Lake Eyre d) flightless bird
5. Jewel of the Earth e) Pointed limestone pillar

Answer:

1. Pinnacle e) Pointed limestone pillar
2. Krill c) small red fish ‘
3. Ostrich d) flightless bird
4. Lake Eyre b) salt lake
5. Jewel of the Earth a) Equatorial forest

IV. Let us learn.
1. Assertion (A) : Aurora is a curtain of colour lights appear in the sky.
Reason (R) : They are caused by magnetic storms in the upper atmosphere.
a) Both A and R are individually true and R is the correct explanation of A.
b) Both A and R are individually true but R is not the correct explanation for A.
c) A is true but R is false.
d) R is true but A is false
Answer:
a) Both A and R are individually true and R is the correct explanation of A.

Samacheer Kalvi 9th Social Science Guide Geography Chapter 7 Mapping Skills

2. Assertion (A) : A geological feature of Africa is the Great Rift Valley.
Reason (R) : A Rift valley is a large crack in the earth’s surface formed by tectonic activity.
a) Both A and R is individually true and R is the correct explanation for A.
b) Both A and R are individually true but R is not the correct explanation for A.
c) A is true but R is false.
d) R is true but, A is false.
Answer:
a) Both A and R is individually true and R is the correct explanation for A

V. Answer in brief.
1. Why Africa is called a “Mother Continent”?
Answer:
Africa is nicknamed the ‘Mother Continent’ as it was the oldest inhabited continent on Earth.

2. What are the important rivers of Africa?
Answer:
The most important rivers in Africa

  • River Nile – father of African rivers.
  • River Congo or Zaire – the second largest river.
  • River Niger – Major river.
  • River Zambezi – fourth largest river and River of life.

3. Name the physical division of Australia.
Answer:
The Physical divisions of Australia are –

  1. The Great Western Plateau
  2. The Central Low lands
  3. The Eastern High lands

Samacheer Kalvi 9th Social Science Guide Geography Chapter 7 Mapping Skills

4. Write about the nature of the Antarctic continent.
Answer:

  • Antarctica is the southernmost and fifth-largest continent in the world.
  • As it is located in polar regions it is the coldest continent with a permanent cover of ice.
  • Its landform consists of mountains, peaks, valleys, glaciers, and plateau.

5. Mention any four economic activities of Australia.
Answer:
Agriculture, forestry, fishing mining, manufacturing, trade, and services are the major economic activities of Australia.

VI. Distinguish between.
1. Sahel and Sahara

Sahel Sahara
Sahel means border or margin Largest hot desert in the world.
It is largely a semi – acid belt of barren, sandy and rocky land. It covers the area of 11 countries.
This region marks the physical and cultural transition. Mt. Koussi, an extinct volcano in chad is the highest point in Sahara.

2. Western Antarctica and Eastern Antarctica.

Western Antarctica Eastern Antarctica
Faces the Pacific Ocean. Faces the Atlantic and Indian Ocean.
Antarctica Penninsula which points towards South America shows that it is the continuation of the Andes mountain range. Mt. Erebus is an active volcano. Located in Ross Island. Onlycontinent called white continent because of the ice cap in 4,000 meters deep.

3. Great Barrier Reef and the Artesian Basin.

Barrier Reef Artesian Basin
1. It is located in the northeast of Australia along the east coast of Queensland in the Pacific Ocean. The Artesian Basin is a region on the earth’s surface where water gushes out like a fountain.
2. It is formed by the tiny coral polyps. Largest and deepest basin in the world.
3. It is about 2300 km long. Located in the west of the Great Dividing range.
4. One of the natural wonders of the world. Found in arid and Semi-arid parts of Queensland.

VII. Give reasons.
1. Egypt is called the gift of the Nile.
Answer:

  • The Nile is the lifeline of Egypt
  • Without the Nile, Egypt would have been a desert
  • So Egypt in the gift of the Nile.

2. Deserts are found in the western margins of the continents.
Answer:

  • Most deserts are found in the Western margins of the subtropics.
  • Because the prevailing winds in the tropics are tropical easterly winds.
  • The tropical easterly winds become dry by the time they reach the western margins.

Samacheer Kalvi 9th Social Science Guide Geography Chapter 7 Mapping Skills

3. Antarctica is called the continent of scientists.
Answer:

  • Scientists of any country are free to conduct experiments and collect data from Antarctica.
  • Hence it is called the ‘continent of Science’.

VIII. Answer in a paragraph each.
1. Give an account of the mineral wealth of Australia.
Answer:

  • Minerals are the largest export item in Australia.
  • It contributes about 10 % of the country’s GDP.
  • Australia is the world’s leading producer of bauxite, limonite, the second-largest producer of gold. lead, lithium, manganese ore, and zinc.
  • The third-largest producer of iron ore and uranium and the fourth-largest producer of black coal.
  • The Coal belts of the country stretch from New castle to Sydney.
  • Iron ores are found mainly in southern and Western Australia.
  • Bauxite is mined around the Gulf of Carpentaria.
  • Uranium is mined in northern territory.
  • Gold is mined in the Western desert at Kalgoorlie and Coolgardie.

2. Describe the flora and fauna of Antarctica.
Answer:
Flaora and Fauna:

  1. Since the temperature is below freezing point almost throughout the year, no major vegetation is found in this continent.
  2. Simple plants like algae, mosses, liverworts, lichens, and microscopic fungi can survive and grow in Antarctica.
  3. Some algae live in the snow, while other plants grow on the coastal rocky land that is ice-free.
  4. A few species of plants, such as plankton, algae, and mosses are seen in and around, Antarctica’s fresh and saltwater lakes.
  5. Small redfish called krill are found in large shoals. It is food for many warm-blooded sea animals.
  6. The blue whale is the largest animal that feeds on plankton. All these animals and birds have a thick layer of fat called blubber which helps them to withstand the cold condition.
  7. Penguin birds in Antarctica cannot fly. They have webbed feet and flippers instead of wings. Small invertebrates are the only land animals which live in the continent.

3. Name the physical divisions of Africa and explain anyone.
Answer:
The physical divisions of Africa are

  1. Sahara
  2. Sahel
  3. Savanna
  4. The Great Rift Valley and the Great Lakes of Africa
  5. East Africa Highlands
  6. Swahili Coast
  7. The Congo Basin or Zaire Basin
  8. Southern Africa.

Sahel:

  • Sahel means border or margin. Sahel is a semi-arid tropical Savannah region that lies between Sahara.
  • Desert in the north and grassland into the south.
  • It Stretches east-west for a distance of 4000 km and covers an area of 30 million sq Km.
  • It is largely a semi-arid belt of barren, sandy, and rocky land.
  • This region marks the physical and cultural transition between the more fertile tropical regions in the south and desert in the north.

X. Activity.
1. Find out the hemisphere and season during December for the following countries

Country Hemisphere Season
South Africa Southern Summer
Morocco Northern Winter
Australia Southern Summer
Niger Northern Winter
Egypt’ Northern Winter
Tasmania Southern Summer
India Northern Winter

Samacheer Kalvi 8th Social Science Exploring Continents Africa, Australia and Antarctica Additional Important Questions and Answers

I. Choose the correct answer.
1. Jasmania is also known as
a) Apple Island
b) Bourke
c) Artesion Basin
d) Sydney
Answer:
a) Apple Island

2. …………….. was discovered by Captian James Cook in 1770
a) Canberra
b) Australia
c) Africa
d) Antarctica
Answer:
b) Australia

Samacheer Kalvi 9th Social Science Guide Geography Chapter 7 Mapping Skills

3. Tropical rain forest is called the
a) Tropical deserts
b) Jewel of the earth
c) Mother continent
d) Rift Valley
Answer:
b) Jewel of the earth

4. Lake is the largest freshwater body in Africa.
a) Supervisor
b) Victoria
c) Albert
d) Kiva
Answer:
b) Victoria

5. Mount an extinct volcano in chad is the highest point in the Sahara
a) Koussi
b) Kilimanjaro
c) Everest
d) Atlas
Answer:
a) Koussi

6. …………………. is nicknamed the ”mother continent”.
a) Africa
b) Australia
c) South Africa
d) None of these
Answer:
a) Africa

7. the Maghreb means west in language.
a) greek
b) Sanskrit
c) Arabic
d) None of these
Answer:
c) Arabic

8. …………….. is one of the largest hot deserts in the world.
a) Sahara
b) Sahel
c) Savanna
d) Great Rift Valley
Answer:
a) Sahara

9. …………….. flows through West-Central Africa and drains into the Atlantic Ocean.
a) River Nile
b) River Congo
c) River Niger
d) River
Answer:
b) River Congo

Samacheer Kalvi 9th Social Science Guide Geography Chapter 7 Mapping Skills

10. The Tropic of Capricorn cuts the continent almost into equal halves.
a) three
b) three
c) four
d) two
Answer:
d) two

II. Fill in the blanks.
1. …………… is the largest research station on Antarctica.
Answer:
Mcmurdo

2. Vinson. …………….. is the highest peak in Antarctica.
Answer:
Massif

3. Wool is described as ……………. of Australia.
Answer:
Cash Crop

Samacheer Kalvi 9th Social Science Guide Geography Chapter 7 Mapping Skills

4. …………… is the longest river in Australia.
Answer:
River Murray

5. Great Barrier Reef is formed by …………………
Answer:
Coral Polyps

6. Sahel means ………………..
Answer:
border or margin

7. …………… is the capital of ghana.
Answer:
Acera

8. …………… was the first to use the term “Dark Continent”.
Answer:
Henry M. Stanly

9. ………….. is the deepest point in Sahara.
Answer:
The Qattara Depression

10. …………… is the highest point in Atlas Mountain.
Answer:
Mount Jaubbakal

11. ………….. is one of the largest plains in Savanna.
Answer:
The Serengeti Plain

12. People of Swahili culture is called ……………….
Answer:
Swahili

13. ………………… Mountain is found in the eastern portion of the escarpment.
Answer:
Drakensberg

14. Nile river is also called ………………
Answer:
Father of African river

15. The country Egypt is called ………………….
Answer:
gift of the Nile

III. Match the following.

1. Bamboo tree a) Veld
2. Karoo b) Libya
3. Open air Zoo c) Africa
4. A1 – ziza d) Sheep rearing
5. Grassland e) Savanna

Answer:

1. Bamboo tree c) Africa
2. Karoo d) Sheep rearing
3. Open air Zoo e) Savanna
4. A1 – ziza b) Libya
5. Grassland a) Veld

IV. Let us learn.
1. Assertion (A): Antarctica is a unique continent but it does not have a native population. There is no country in Antarctica.
Reason (R): It is located in the polar region, it is the coldest continent with a permanent cover of ice.
a) Both A and R are individually true and R is the correct explanation for A.
b) Both A and R are individually true but R is not the correct explanation for A.
c) A is true but R is false.
d) R is true but A is false.
Answer:
a) Both A and R are individually true and R is the correct explanation for A.

Samacheer Kalvi 9th Social Science Guide Geography Chapter 7 Mapping Skills

2. Assertion (A): A rift valley is a large crack in the earth’s surface formed by the shifting of tectonic plates.
Reason (R): It runs through eastern Africa and contains many lakes.
a) Both A and R are individually true and R is the correct explanation for A.
b) Both A and R are individually true but R is not the correct explanation for A.
c) A is true but R is false.
d) R is true but A is false.
Answer:
a) Both A and R are individually true and R is the correct explanation for A.

V. Answer in brief.
1. Write a short note on Xerophytes.
Answer:
The Plants and trees in Australia are adapted to dry conditions and can survive for long period without water called Xerophytes.

2. What is the Political division of Australia?
Answer:

  • The political division of Australia are six states and two union territories.
  • They are New South Wales, Queensland, South Australia, Jasmania, Victoria, Western Australia, Northern Territory, and Capital Territory.

3. What do you know about the “Continent of Science”?
Answer:
Scientists of any Country are free to conduct experiments and collect data from Antarctica and so it is called the continent of science.

4. Great Barrier Reef – Explain
Answer:

  • Great Barrier Reef is located in the northeast of Australia along the east coast of Queen’s land in the Pacific Ocean.
  • It is formed by the tiny coral polyps.

Samacheer Kalvi 9th Social Science Guide Geography Chapter 7 Mapping Skills

5. State a few minerals found in Antarctica.
Answer:
Few minerals found in Antarctica are gold, Platinium, Nickel, Copper, and petroleum.

VI. Distinguish between.
1. The Great Western Plateau and The eastern highlands.

The Great Western Plateau (Australia) The eastern highlands
1. It is Australia’s largest physical division. Its area is about 2,700,000 square kilometers. It extends for about 3860 km along the eastern edge of Australia.
2. It is 863 meters high above sea level. It is one of the natural wonders of Australia. Australian Alps mountain range is the highest mountain range in Australia.
It is covered with ice.
3. The Great Victoria Desert is the largest desert in Australia located in Western, Australia, and South Australia. The highest peak of this range is Mt. Kosciuszko (2230 m) and is located in New South Wales.

2. River Nile and River Congo or Zaire.

River Nile River Congo or Zaire
The Nile is the longest river in the world. The Nile is known as the “Father of African Rivers”. Congo is the second largest river in Africa after the Nile.
Its length is about 6650 km. Its length is about 4700km.
It flows northward and drains into the Mediterranean-sea. It flows through West-Central Africa and drains into the Atlantic Ocean.

VII. Give reasons.
1. The Zambezi River is a Southern African “River of Life.
Answer:

  • The fourth-largest river system in Africa.
    It drains seven countries and supports millions of people, who make use of its rich fisheries, forests, water, and rich floodplain soils.

2. the Nile is known as the “Father of African Rivers”.

  • Nile River the longest river in the world, called the father of African rivers.
  • It rises south of the Equator and flows northward through northeastern Africa to drain into the Mediterranean Sea.
  • It has a length of about 6,650 kilometers and drains an area estimated at 3,349,000 square kilometers.

1. How is the climatic condition of Africa divided?
Answer:
It is divided into six major climatic zones. They are.
Arid and semi-arid climate:

  • Northern & Southern Africa
  • Rainfall – scanty Tropical Savanna Climate:
  • 100-200 latitude or either side of the equator
  • The tropical wet and dry climate
  • Equatorial climate
  • The equatorial region near the congo basin and east African highlands.
  • Temperature and rainfall are high.

Temperature Climate:

  • The southern tip of southern Africa.
  • Climate Equable
  • Mediterranean Climate
  • NorthWestern and Southwestern tip of Africa.
  • Gets rainfall in winter while in summer it is hot and dry.

Tropical Monsoon Climate:

  • Eastern shore of Africa.
  • Summer is hot with monsoon winds bringing good rainfall winter is cool and dry.

2. What’s the role of agriculture in Africa?
Answer:

  • Agriculture is a major economic activity.
  • Wheat is grown in temperate grasslands.
  • Rice is cultivated on the Guinea coast.
  • Maize and Millets are grown all over the plateau.
  • Cotton is the chief cash crop of Africa.
  • Coffee is grown in Ethiopia
  • Ghana is the chief producer of cocoa.
  • Oil palm in west African countries.
  • Sugarcane, rubber, tobacco are the major crops of East African countries.

Samacheer Kalvi 8th Maths Guide Chapter 7 Information processing Ex 7.3

Tamilnadu State Board New Syllabus Samacheer Kalvi 8th Maths Guide Pdf Chapter 7 Information processing Ex 7.3 Text Book Back Questions and Answers, Notes.

Tamilnadu Samacheer Kalvi 8th Maths Solutions Chapter 7 Information processing Ex 7.3

Question 1.
Fill in the blanks (Use Atbash Cipher that is given in code 3)
Hint:
For this question, we need to use Atbash cipher. For Atbash cipher, first we write the alphabets from A to Z and then in reverse from Z to A below that.
A B C D E F G H I J K L M N O P Q R S T U V W X Y Z
Z Y X W V U T S R Q P O N M L K J I H G F E D C B A

Samacheer Kalvi 8th Maths Guide Chapter 7 Information processing Ex 7.3

(i) G Z N R O = _________
Answer:
TAMIL
Hint:
Now to solve, we look up the corresponding letter from the table to replace in code to get the actual word.
So, for G Z N R O, from table,
for G, it is T
for Z, it i sA
for N, it is M
for R, it is i
for 0, it is L
So, the actual word is TAMIL

(ii) V M T O R H S = ________
Answer:
ENGLISH

Hint:
VMTORHS
To solve, we look up the corresponding letter from table to replace in code to get the actual word.
For V, it is E
for M, it is N
for T, it is G
for O, it is L
for R, it is I,
for H, it is S
for S, it is H
Therefore we get E N G L I S H = ENGLISH

Samacheer Kalvi 8th Maths Guide Chapter 7 Information processing Ex 7.3

(iii) N Z G S V N Z G R X H _________
Answer:
MATHEMATICS
Hint:
Similarly as above for
Samacheer Kalvi 8th Maths Guide Answers Chapter 7 Information Processing Ex 7.3 1

(iv) H X R V M X V __________
Answer:
SCIENCE
Hint:
For
Samacheer Kalvi 8th Maths Guide Answers Chapter 7 Information Processing Ex 7.3 2

(v) H L X R Z O H X R V M X V = __________
Answer:
SOCIAL SCIENCE
Hint:
Samacheer Kalvi 8th Maths Guide Answers Chapter 7 Information Processing Ex 7.3 3

Samacheer Kalvi 8th Maths Guide Chapter 7 Information processing Ex 7.3

Question 2.
Match the following (a = 00 ………… Z = 25).
Samacheer Kalvi 8th Maths Guide Answers Chapter 7 Information Processing Ex 7.3 4
Answer:
(i) – c
(ii) – d
(iii) – a
(iv) – e
(v) – b
Hint:
(i) Mathematics is
Samacheer Kalvi 8th Maths Guide Answers Chapter 7 Information Processing Ex 7.3 5
So matching option is c

(ii) addition is
Samacheer Kalvi 8th Maths Guide Answers Chapter 7 Information Processing Ex 7.3 6
Matching option is d

Samacheer Kalvi 8th Maths Guide Chapter 7 Information processing Ex 7.3

(iii) Subtraction is
Samacheer Kalvi 8th Maths Guide Answers Chapter 7 Information Processing Ex 7.3 7
multiplication is a

(iv) multiplication is
Samacheer Kalvi 8th Maths Guide Answers Chapter 7 Information Processing Ex 7.3 8
Matching option is e
however instead of 25, it should be 08

(v) division is
Samacheer Kalvi 8th Maths Guide Answers Chapter 7 Information Processing Ex 7.3 9
Matching option is b

Question 3.
Frame Additive cipher table (key = 4).
Answer:
Step 1 : write all alphabets
Step 2 : Assign numbers to each alphabet starting from 00 till 25.
Step 3 : add key value (here it is 4) to the numbers assigned in step 2 to form cipher table
Samacheer Kalvi 8th Maths Guide Answers Chapter 7 Information Processing Ex 7.3 10

Samacheer Kalvi 8th Maths Guide Chapter 7 Information processing Ex 7.3

Question 4.
A message like “Good Morning” written in reverse would instead be “Doog Gninrom” In the same way decode the sentence given below:
“Ot dnatsrednu taht scitamehtam nac eb decneirepxe erehwreve ni erutan dna laer efil”
Answer:
Given that good morning written in reverse is doog gninrom.
We have to decode the below by reversing, so,
Ot dnatsrednu taht scitamehtam nac eh decneirepxe erehwreve ni erutan dna laer efil.

Ans: to understand that mathematics can be experienced everywhere in nature and real life.

Question 5.
Decode the given Pigpen Cipher text and compare your answer to get result.

Samacheer Kalvi 8th Maths Guide Answers Chapter 7 Information Processing Ex 7.3 12
I. The room number in which the treasure took place

Answer:
The room in which the treasure took place = 28

II. Place of the treasure

Answer:
The place of treasure = Chair

III. The name of the treasure
Samacheer Kalvi 8th Maths Guide Answers Chapter 7 Information Processing Ex 7.3 15
Answer:
Identity of treasure = Gift voucher.

Samacheer Kalvi 8th Maths Guide Chapter 7 Information processing Ex 7.3

Question 6.
Praveen recently got the registration number for his new two-wheeler. Here, the number is given in the form of mirror-image. Encode the image and find the correct registration number of praveen’s two-wheeler.
T N 1 2 H 2 5 8 9
(a)
Samacheer Kalvi 8th Maths Guide Answers Chapter 7 Information Processing Ex 7.3 16
(b)
Samacheer Kalvi 8th Maths Guide Answers Chapter 7 Information Processing Ex 7.3 17
(c)
Samacheer Kalvi 8th Maths Guide Answers Chapter 7 Information Processing Ex 7.3 18
(d)
Samacheer Kalvi 8th Maths Guide Answers Chapter 7 Information Processing Ex 7.3 19
Answer:
The mirror image is
Samacheer Kalvi 8th Maths Guide Answers Chapter 7 Information Processing Ex 7.3 18
When we place an imaginary mirror & visualize the image seen in the mirror, we will get the below.
Samacheer Kalvi 8th Maths Guide Answers Chapter 7 Information Processing Ex 7.3 20
∴ The answer option c

Samacheer Kalvi 8th Maths Guide Chapter 7 Information processing Ex 7.3

Objective Type Questions

Question 7.
In questions (i) and (ii), there are four groups of letters in each set. Three of these sets are alike in some way while one is different. Find the one which is different.
(i).(A) C R D T
(B) A P B Q
(C) E U F V
(D) G W H X
Answer:
(i) (A) C R D T
Hint:
The four groups of letters are
CRDT APBQ EUFV GWHX
The above can be written as
Samacheer Kalvi 8th Maths Guide Answers Chapter 7 Information Processing Ex 7.3 21
We find that when we take 1st & 3rd letter & 2nd & 4th letter as 2 pairs, the 3 letter is the next letter alphabetically to the 1st letter.
Similarly the 4th letter is alphabetically the next letter of the 2nd letter.
i.e CD, AB, EF, GH & PQ, UV, WX
Only in CRDT, we have T instead of ‘S’
So, Ans: in CRDT ⇒ Option (a)

Samacheer Kalvi 8th Maths Guide Chapter 7 Information processing Ex 7.3

(ii). (A) H K N Q
(B) I L O R
(C) J M P S
(D) A D G J
Answer:
(D) A D G J
Hint:
The four groups of letters are
HKNQ ILOR IMPS ADGI
If we notice, we find that 2 letters are missing in the sequence. le.
HIJ KLI NOP Q
IJK LMN OPQ R
JKL MNO PQR S
ABC DEF GH I
We find that only in ADGI, the difference is only one letter between G & I.
Hence it is the odd one out.

Samacheer Kalvi 8th Maths Guide Chapter 7 Information processing Ex 7.3

Question 8.
A group of letters are given. A numerical code has been given to each letter. These letters have to be unscrambled into a meaningful word. Find out the code for the word so formed from the 4 answers given.
Samacheer Kalvi 8th Maths Guide Answers Chapter 7 Information Processing Ex 7.3 22
(A) 2 3 4 1 5 6
(B) 5 6 3 4 2 1
(C) 6 1 3 5 2 4
(D) 4 2 1 3 5 6
Answer:
(B) 5 6 3 4 2 1

Hint:
Given code is
Samacheer Kalvi 8th Maths Guide Answers Chapter 7 Information Processing Ex 7.3 22
Option (a) is 234156. When we substitute number for each letter from code, we get,
Samacheer Kalvi 8th Maths Guide Answers Chapter 7 Information Processing Ex 7.3 23

Option (b) is 563421, similarly, we get
Samacheer Kalvi 8th Maths Guide Answers Chapter 7 Information Processing Ex 7.3 24

Option (c) is
Samacheer Kalvi 8th Maths Guide Answers Chapter 7 Information Processing Ex 7.3 25

Option d) is
Samacheer Kalvi 8th Maths Guide Answers Chapter 7 Information Processing Ex 7.3 26

So, only in option b, we get a meaningful word, i.e PENCIL.
Hence, answer is Option b.

Samacheer Kalvi 8th Maths Guide Chapter 7 Information processing Ex 7.3

Question 9.
Questions (iii) and (iv) are based on code language. Find the correct answer from the four alternatives given.
(iii)Inacertain code, ‘M E D I C I N E’ is coded as ‘E O J D J E F M’,then how is ‘C O M P U T E R’ written in the same code?
(A) C N P R V U F Q
(B) C M N Q T U D R
(C) R F I J V Q N P C
(D) R N V F T U D Q
Answer:
(C) R F I J V Q N P C
Hint:
lt is given that in a certain code MEDlClNE is coded as E O J D J E F M
When we observe the word & the code, we find that, there is a pattern.
Samacheer Kalvi 8th Maths Guide Answers Chapter 7 Information Processing Ex 7.3 27
[to understand, see the matching shapes]
To get the code from the word, we follow the below steps

1.
Samacheer Kalvi 8th Maths Guide Answers Chapter 7 Information Processing Ex 7.3 28
E [E D I C I N] M

2. For the middle letters, replace the letters with their alphabetically next letters, so we get
Samacheer Kalvi 8th Maths Guide Answers Chapter 7 Information Processing Ex 7.3 29

3. Now we have to reverse the order of the middle letters in the bracket, so we get
E [O J D J E F] M = E O J D J E F M
Thus we get the code
So similarly, we have to follow the 3 steps to get code, therefore:
Samacheer Kalvi 8th Maths Guide Answers Chapter 7 Information Processing Ex 7.3 30
Step 1: Swap 1st & last letters, so we get
R [O M P U T E] C

Step 2: For the middle letters, replace the letters with alphabetically next letters, so we get
R [P N Q V L F] C

Step 3: Reverse the word of letters in the bracket to we get
R [F U V Q N P] C
∴ Ans is R F U V Q N P C ⇒ option c

Samacheer Kalvi 8th Maths Guide Chapter 7 Information processing Ex 7.3

(iv) lf the word ’P H O N E’ is coded as ’S K R Q H’, how will ’R A D I O’ be coded?
(A) S C G N H
(B) V R G N G
(C) U D G L R
(D) S D H K Q
Answer:
(C) U D G L R
Hint:
If PHONE is coded as S K R Q H
Find that code for R A D I O

We find that we get the code, by 3 letter alphabetically so. from P. skipping Q & R. we get S
Samacheer Kalvi 8th Maths Guide Answers Chapter 7 Information Processing Ex 7.3 31
Similarly, from H, skipping I & J, we get K
Like wise for R A D I O. skipping the 2 alphabets,
From R skip S & T → U
From A skip B & C → D
From D skip E & F → G
From l skip J & K → L
From O skip P & Q → R
Samacheer Kalvi 8th Maths Guide Answers Chapter 7 Information Processing Ex 7.3 32
∴ Ans is UDGLR ⇒ option c

Samacheer Kalvi 8th Maths Guide Chapter 7 Information processing Ex 7.1

Tamilnadu State Board New Syllabus Samacheer Kalvi 8th Maths Guide Pdf Chapter 7 Information processing Ex 7.1 Text Book Back Questions and Answers, Notes.

Tamilnadu Samacheer Kalvi 8th Maths Solutions Chapter 7 Information processing Ex 7.1

Question 1.
You want to have an ice cream or a cake. There are three flavours (chocolate, strawberry and vanilla) in ice creams, and two flavours (orange and red velvet) in the cakes. In how many possible ways can you choose an ice cream or a cake?
Samacheer Kalvi 8th Maths Guide Answers Chapter 7 Information Processing Ex 7.1 1
Answer:
We are going to have either a ice cream or a cake.
Ice cream can be selected from 3 flavors and cake from two flavors. Both the events cannot
occur simultaneously selecting ice cream and cake.
∴ Number of possible ways = 3 + 2
= 5 ways

Samacheer Kalvi 8th Maths Guide Chapter 7 Information processing Ex 7.1

Question 2.
Shanthi has 5 chudithar sets and 4 frocks. In how many possible ways, can she wear either a chudithar or a frock?
Answer:
Shanthi her 5 chudidhar sets and 4 frocks.
She wear either chudidhar or a frock.
∴ Total possible ways = 5 + 4 = 9 ways

Question 3.
In a Higher Secondary School, the following groups are available in XI standard
I. Science Group:
(i) Physics, Chemistry, Biology and Mathematics
(ii) Physics, Chemistry, Mathematics and Computer Science.
(iii) Physics, Chemistry, Biology and Home Science

Il. Arts Group:
(i) Accountancy, Commerce, Economics and Business Maths
(ii) Accountancy, Commerce, Economics and Computer Science
(iii) History, Geography, Economics and Commerce

III. Vocational Group:
(i) Biology, Nursing Theory, Nursing Practical I and Nursing Practical II
(ii) Home Science, Textiles and Dress Designing Theory, Textiles and Dress
Designing Practical I and Textiles and Dress Designing Practical Il
In how many possible ways, can a student choose a group?
Answer:
The student either select any one of science group in 3 ways or any of the arts group in 3
ways or any of the vocational group in 2 ways.
∴ Total possible ways = 3 + 3 + 2 = 8 ways

Samacheer Kalvi 8th Maths Guide Chapter 7 Information processing Ex 7.1

Question 4.
If you have 2 school bags and 3 water bottles then, in how many different ways can you choose each one of them, while going to school?
Answer:
We can select one school bag from 2 and one bottle from 3 as follows.
Samacheer Kalvi 8th Maths Guide Answers Chapter 7 Information Processing Ex 7.1 2
∴ A bag and a water bottle can be selected in 2 × 3 = 6 ways.

Question 5.
Roll numbers are created with a letter followed by 3 digits in it, from the letters A, B, C, D and E and any 3 digits from 0 to 9. In how many possible ways can the roll numbers be generated? (except A000, B000, C000, D000 and E000)
Answer:
We have a letter followed by 3 digits in the roll number.
The letter is selected from the five letters A, B, C, D, E.
For these 5 letters we have to select a 3 digit number using the digits O to 9.
Ones place can be formed using any one of the 10 number 0 to 9 in 10 ways.
Tens place can be formed in I O ways.
∴ A two digit number can be formed in 10 × 10 = 100 ways.
Thousands place can be formed in lo ways
∴ A 3 digit number can be formed in 10 × 10 × 10 = 1000 ways.
∴ 5 letters can be attached in 5 × 1000 = 5000 ways.
∴ The roll number can be formed in 5000 – 5 = 4995 ways.

Samacheer Kalvi 8th Maths Guide Chapter 7 Information processing Ex 7.1

Question 6.
A safety locker in a jewel shop requires a 4 digit unique code. The code has the digits from 0 to 9. How many unique codes are possible?
Samacheer Kalvi 8th Maths Guide Answers Chapter 7 Information Processing Ex 7.1 3
Answer:
The unique code has 4 digits.
Each digit is formed using any of the 10 numbers from 0 to 9.
∴ Single digit number can be formed in 10 ways.
A double digit number can be formed in 10 × 10 ways.
A three digit number can be formed in 10 × 10 × 10 ways.
A four digit number can be formed in 10 × 10 × 10 × 10 ways. = 10,000 ways

Samacheer Kalvi 8th Maths Guide Chapter 7 Information processing Ex 7.1

Question 7.
An examination paper has 3 sections, each with five questions and students are instructed to answer one question from each section. In how many different ways of can the questions be answered?
Answer:
The tree diagram for this may be
Samacheer Kalvi 8th Maths Guide Answers Chapter 7 Information Processing Ex 7.1 4
∴ Number of possible ways to select one questions from each of 3 sections is 3 × 5 = 15 ways

Samacheer Kalvi 8th Maths Guide Chapter 7 Information processing Ex 7.1

Question 8.
The given spinner is spun twice and the two numbers got are used to form a 2 digit number. How many different 2 digits numbers are possible?
Samacheer Kalvi 8th Maths Guide Answers Chapter 7 Information Processing Ex 7.1 5
Answer:
On the first spin we get any of the five numbers to form ones place then insecond spin the number got will fill 10’s place.
∴ Number of ways = 5 × 5 = 25 ways.
Removing the repetitions (11, 22, 33, 44, 55) once we get 25 – 5 = 20 ways.
20 different two digit numbers are possible

Question 9.
Ramya wants to paint a pattern in her living room wall with minimum budget. Help her to colour the pattern with 2 colours but make sure that no two q adjacent boxes are the same colour. The pattern is shown in the picture.
Samacheer Kalvi 8th Maths Guide Answers Chapter 7 Information Processing Ex 7.1 6
Answer:
Samacheer Kalvi 8th Maths Guide Answers Chapter 7 Information Processing Ex 7.1 7

Samacheer Kalvi 8th Maths Guide Chapter 7 Information processing Ex 7.1

Question 10.
Colour the regions in the maps with few colours as possible but make sure that no two adjacent countries are of the same colour.
Samacheer Kalvi 8th Maths Guide Answers Chapter 7 Information Processing Ex 7.1 8
Answer:
Samacheer Kalvi 8th Maths Guide Answers Chapter 7 Information Processing Ex 7.1 9

Objective Type Questions

Question 11.
In a class there are 26 boys and 15 girls. The teacher wants to select a boy or a girl to represent a quiz competition. In how many ways can the teacher make this selection?
(A) 41
(B) 26
(C) 15
(D) 390
Answer:
41

Question 12.
How many outcomes can you get when you toss three coins once?
(A) 6
(B) 8
(C) 3
(D) 2
Answer:
8

Samacheer Kalvi 8th Maths Guide Chapter 7 Information processing Ex 7.1

Question 13.
In how many ways can you answer 3 multiple choice questions, with the choices A,B,C and D?
(A) 4
(B) 3
(C) 12
(D) 64
Answer:
64

Question 14.
How many 2 digit numbers contain the number 7?
(A) 10
(B) 18
(C) 19
(D) 20
Answer:
18